Tag Archives: 2015

ĐỀ THI VÀO LỚP 10 CHUYÊN TOÁN TP.HCM – NĂM 2015

Bài 1. Cho hai số thực $a, b$ thỏa điều kiện $a b=1, a+b \neq 0$. Tính giá trị của biểu thức:

$P=\frac{1}{(a+b)^{3}}\left(\frac{1}{a^{3}}+\frac{1}{b^{3}}\right)+\frac{3}{(a+b)^{4}}\left(\frac{1}{a^{2}+b^{2}}\right)+\frac{6}{(a+b)^{5}}\left(\frac{1}{a}+\frac{1}{b}\right)$

Bài 2. (a) Giải phương trình: $2 x^{2}+x+3=3 x \sqrt{x+3}$

(b) Chứng minh rằng: $a b c\left(a^{3}-b^{3}\right)\left(b^{3}-c^{3}\right)\left(c^{3}-a^{3}\right)$ chia hết cho 7 với mọi số nguyên $a, b, c$

Bài 3. Cho hình bình hành $A B C D$. Đường thẳng qua $C$ vuông góc với $C D$ cắt đường thẳng qua $A$ vuông góc với $B D$ tại $F$. Đường thẳng qua $B$ vuông góc với $A B$ cắt đường trung trực của $A C$ tại $E$. Hai đường thẳng $B C$ và $E F$ cắt nhau tại $K$. Tính tỉ số: $\frac{K E}{K F}$.

Bài 4. Cho hai số dương $a, b$ thỏa mãn điều kiện: $a+b \leq 1$. Chứng minh rằng: $a^{2}-$ $\frac{3}{4 a}-\frac{a}{b} \leq-\frac{9}{4}$

Bài 5. Cho tam giác $\triangle A B C$ có ba góc nhọn nội tiếp đường tròn $(\mathrm{O})$. Gọi $M$ là trung điểm của cạnh $B C$ và $N$ là điểm đối xứng của $M$ qua $O$. Đường thẳng qua $A$ vuông góc với $A N$ cắt đường thẳng qua $B$ vuông góc với $B C$ tại $D$. Kẻ đường kính $A E$.

(a) Chứng minh rằng: $B A \cdot B C=2 B D \cdot B E$

(b) $C D$ đi qua trung điểm của đường cao $A H$ của tam giác $\triangle A B C$.

Bài 6. Mười vận động viên tham gia cuộc thi đấu quần vợt. Cứ hai người trong họ chơi với nhau đúng một trận. Người thứ nhất thắng $x_{1}$ trận và thua $y_{1}$ trận, người thứ hai thắng $x_{2}$ và thua $y_{2}$ trận,… người thứ mười thắng $x_{10}$ trận và thua $y_{10}$. Biết rằng trong một trận đấu quần vợt không có kết quả hòa. Chứng minh rằng:

$x_{1}^{2}+x_{2}^{2}+\ldots+x_{10}^{2}=y_{1}^{2}+y_{2}^{2}+\ldots+y_{10}^{2}$

 

LỜI GIẢI

 

Bài 1. Cho hai số thực $a, b$ thỏa điều kiện $a b=1, a+b \neq 0$. Tính giá trị của biểu thức:

$P=\frac{1}{(a+b)^{3}}\left(\frac{1}{a^{3}}+\frac{1}{b^{3}}\right)+\frac{3}{(a+b)^{4}}\left(\frac{1}{a^{2}+b^{2}}\right)+\frac{6}{(a+b)^{5}}\left(\frac{1}{a}+\frac{1}{b}\right)$

Lời giải. Ta có: $a b=1$ và $a+b \neq 0$

$P=\frac{1}{(a+b)^{3}}\left(\frac{1}{a^{3}}+\frac{1}{b^{3}}\right)+\frac{3}{(a+b)^{4}}\left(\frac{1}{a^{2}}+\frac{1}{b^{2}}\right)+\frac{6}{(a+b)^{5}}\left(\frac{1}{a}+\frac{1}{b}\right)$ $=\frac{1}{(a+b)^{3}}\left(\frac{a^{3} b^{3}}{a^{3}}+\frac{a^{3} b^{3}}{b^{3}}\right)+\frac{3}{(a+b)^{4}}\left(\frac{a^{2} b^{2}}{a^{2}}+\frac{a^{2} b^{2}}{b^{2}}\right)+\frac{6}{(a+b)^{5}}\left(\frac{a b}{a}+\frac{a b}{b}\right)$ $=\frac{1}{(a+b)^{3}}\left(a^{3}+b^{3}\right)+\frac{3}{(a+b)^{4}}\left(a^{2}+b^{2}\right)+\frac{6}{(a+b)^{5}}(a+b)$ $=\frac{a^{2}-a b+b^{2}}{(a+b)^{2}}+\frac{3(a+b)^{2}-6}{(a+b)^{4}}+\frac{6}{(a+b)^{4}}$ $=\frac{a^{2}-a b+b^{2}}{(a+b)^{2}}+\frac{3}{(a+b)^{2}}=\frac{(a+b)^{2}}{(a+b)^{2}}$

Vậy P=1

Bài 2.

a) Giải phương trình: $2 x^{2}+x+3=3 x \sqrt{x+3}$

b) Chứng minh rằng: $a b c\left(a^{3}-b^{3}\right)\left(b^{3}-c^{3}\right)\left(c^{3}-a^{3}\right)$ chia hết cho 7 với mọi số nguyên $a, b, c$

Lời giải.

a) Điều kiện xác định: $x \geq-3$

$Ta có:  2 x^{2}+x+3=3 x \sqrt{x+3} $

$\Leftrightarrow 2 x^{2}-2 x \sqrt{x+3}-x \sqrt{x+3}+x+3=0 $

$\Leftrightarrow(x-\sqrt{x+3})(2 x-\sqrt{x+3})=0 $

$\Leftrightarrow\left[\begin{array} { l }{ x = \sqrt { x + 3 } ( x \geq 0 ) } \\ { 2 x = \sqrt { x + 3 } ( x \geq 0 ) }\end{array} \Leftrightarrow \left[\begin{array}{l}x^{2}-x-3=0(x \geq 0) \\ 4 x^{2}-x-3=0(x \geq 0)\end{array}\right.\right. $

$\Leftrightarrow\left[\begin{array}{l}x=\frac{1+\sqrt{13}}{2} \\ x=1\end{array}\right.$

b) Ta áp dụng bổ đề sau: Lập phương một số nguyên bất kì khi chia cho 7 đều chỉ có số dư là: $0,1,-1$.

Chứng minh tính chất này ta chỉ cần lập bảng số dư.

Nếu một trong ba số $a, b, c$ chia hết cho 7 , ta có điều cần chứng minh.

Nếu $a, b, c$ không có số nào chia hết cho 7 thì $a^{3}, b^{3}, c^{3}$ chia 7 dư $1,-1$, do đó theo nguyên lý Dirichlet thì có 2 số có hiệu chia hết cho 3, do đó ít nhất một trong các số $a^{3}-b^{3}, b^{3}-c^{3}, c^{3}-a^{3}$ chia hết cho 7 . Từ đó ta có điều cần chứng minh.

Bài 3. Cho hình bình hành $A B C D$. Đường thẳng qua $C$ vuông góc với $C D$ cắt đường thẳng qua $A$ vuông góc với $B D$ tại $F$. Đường thẳng qua $B$ vuông góc với $A B$ cắt đường trung trực của $A C$ tại $E$. Hai đường thẳng $B C$ và $E F$ cắt nhau tại $K$. Tính tỉ số: $\frac{K E}{K F}$.

Lời giải.

Gọi $M$ là giao điểm của $A F$ và $B D$ và $J$ là giao điểm của $A B$ và $O E$.

Ta có các tứ giác $A B E O, C D M F$ nội tiếp. Khi đó $\angle J E A=\angle J B O=180^{\circ}-\angle A B O=$ $180^{\circ}-\angle B D C=\angle A F C$

Và $\angle E J A=90^{\circ}-\angle J A O=\angle A C F$.

Khi đó $\triangle A J E \backsim \triangle A F C \Rightarrow \frac{A J}{A C}=\frac{J E}{C F} \Rightarrow \frac{A C}{C F}=\frac{A J}{J E}$. (1)

Mặt khác $\triangle A J O \backsim \triangle E J B \Rightarrow \frac{A J}{J E}=\frac{A O}{B E}$. (2)

Từ (1) và (2) ta có $\frac{A C}{C F}=\frac{A O}{B E} \Rightarrow \frac{B E}{C F}=\frac{A O}{A C}=\frac{1}{2}$.

Mặt khác $\frac{K E}{K F}=\frac{B E}{C F}=\frac{1}{2}$.

Bài 4. Cho hai số dương $a, b$ thỏa mãn điều kiện: $a+b \leq 1$. Chứng minh rằng: $a^{2}-$ $\frac{3}{4 a}-\frac{a}{b} \leq-\frac{9}{4}$

Lời giải. Do $a>0, b>0$ và $a+b \leq 1$. Ta chứng minh: $a^{2}-\frac{3}{4} a-\frac{a}{1-a} \leq-\frac{9}{4}$

$\Leftrightarrow 4 a^{4}-4 a^{3}+13 a^{2}-12 a+3 \geq 0 $

$\Leftrightarrow(2 a-1)^{2}\left(a^{2}+3\right) \geq 0 ${ (luôn đúng)

Dấu bằng trong bất đẳng thức này xảy ra khi: $a=b=\frac{1}{2}$

Bài 5. Cho tam giác $\triangle A B C$ có ba góc nhọn nội tiếp đường tròn $(\mathrm{O})$. Gọi $M$ là trung điểm của cạnh $B C$ và $N$ là điểm đối xứng của $M$ qua $O$. Đường thẳng qua $A$ vuông góc với $A N$ cắt đường thẳng qua $B$ vuông góc với $B C$ tại $D$. Kẻ đường kính $A E$.

a) Chứng minh rằng: $B A \cdot B C=2 B D \cdot B E$

b) $C D$ đi qua trung điểm của đường cao $A H$ của tam giác $\triangle A B C$.

Lời giải.

a) Chứng minh rằng: $B A \cdot B C=2 B D \cdot B E$

Điều này tương đương với: $B A . B M=B D . B E$

Xét hai tam giác $\triangle B M E$ và tam giác $\triangle B D A$ ta có:

$\angle D B A=\angle M B E$ (cùng phụ với góc $\angle A B C$ ) (1)

$\angle D A B+\angle B A E+\angle O A N=90^{\circ}$

Do $\triangle A O N=\triangle E O M$ nên $\angle O A N=\angle O E M$

Từ đó ta có: $\angle D A B+\angle B A E+\angle O E M=90^{\circ}$

Do $A E$ là đường kính của đường tròn $(O)$. Nên $\triangle A B E$ vuông tại $B$.

Từ đó ta có: $\angle B A E+\angle O E M+\angle B E M=90^{\circ}$

Vậy: $\angle D A B=\angle B E M$ (2)

Từ (1) và (2) ta có $\triangle B M E \backsim \triangle B D A$

Vậy: $\frac{B M}{B D}=\frac{B E}{B A}=\frac{M E}{D A}$ (đpcm)

b) Chứng minh rằng: $C D$ đi qua trung điểm $I$ của $A H$ Gọi $F$ là giao điểm của $B D$ và $C A$

Từ đó điều phải chứng minh tương đương với chứng minh rằng $D$ là trung điểm của $B F$. Mà $M$ là trung điểm của $B C$ như vậy ta chỉ cần chứng minh được: $M D | A C$

Xét hai tam giác vuông $\triangle B D M$ và tam giác $\triangle B A E$ ta có: $\frac{B D}{B M}=\frac{B A}{B E}$ Vậy: $\triangle B D M \backsim \triangle B A E$

Từ đó ta có: $\angle B M D=\angle B E A$

Mà: $\angle B E A=\angle B C A$ (cùng chắn cung $A B$ của đường tròn $(O)$ )

Vậy: $\angle B M D=\angle B C A$

Từ đó ta có: $M D | A C$. Đây cũng chính là điều phải chứng minh.

Bài 6. Mười vận động viên tham gia cuộc thi đấu quần vợt. Cứ hai người trong họ chơi với nhau đúng một trận. Người thứ nhất thắng $x_{1}$ trận và thua $y_{1}$ trận, người thứ hai thắng $x_{2}$ và thua $y_{2}$ trận,… người thứ mười thắng $x_{10}$ trận và thua $y_{10}$. Biết rằng trong một trận đấu quần vợt không có kết quả hòa. Chứng minh rằng:

$x_{1}^{2}+x_{2}^{2}+\ldots+x_{10}^{2}=y_{1}^{2}+y_{2}^{2}+\ldots+y_{10}^{2}$

Lời giải. Do trong một trận đấu chỉ có thắng hoặc thua nên tổng số trận thằng phải bằng với tổng số trận thua. Từ đó ta có:

$x_{1}+\ldots \ldots . .+x_{10}=y_{1}+\ldots \ldots . .+y_{10}$

Ta có tổng cộng là 45 trận $\left(\frac{10.9}{2}\right)$ nên tổng sổ trận thắng bằng tổng số trận thua bằng 45 trận.

$x_{1}+\ldots \ldots . .+x_{10}=y_{1}+\ldots \ldots . .+y_{10}=45$

Mỗi người sẽ thi đấu với 9 người còn lại nên:

$x_{i}+y_{i}=9 \Leftrightarrow x_{i}=9-y_{i} \Leftrightarrow x_{i}^{2}=81-18 y_{i}+y_{i}^{2}$

Từ đó ta có:

$\sum_{i=1}^{10} x_{i}^{2}=810-18 \sum_{i=1}^{10} y_{i}+\sum_{i=1}^{10} y_{i}^{2}=810-18.45+\sum_{i=1}^{10} y_{i}^{2}=\sum_{i=1}^{10} y_{i}^{2}$

Đây chính là điều phải chứng minh

Bài toán này cũng có thể tổng quát lên cho trường hợp $n$ người, phần này dành cho các em tự chứng minh.

 

 

 

 

 

 

 

 

 

 

 

 

 

 

 

 

 

 

 

 

 

ĐỀ THI VÀO LỚP 10 TRƯỜNG PHỔ THÔNG NĂNG KHIẾU 2015

Bài 1. (a) Giải phương trình $\sqrt{2 x-1}+\sqrt{1-2 x^{2}}=2 \sqrt{x-x^{2}}$.

(b) Cho các số $a$ và $b$ thỏa mãn điều kiện $\sqrt[3]{a}+\sqrt[3]{b}=\sqrt[3]{b-\frac{1}{4}}$. Chứng minh rằng $-1 \leq a<0$.

Bài 2. (a) Tìm các số nguyên $a, b, c$ sao cho $a+b+c=0$ và $a b+b c+a c+3=0$.

(b) Cho $m$ là số nguyên. Chứng minh rằng nếu tồn tại các số nguyên $a, b, c$ khác 0 sao cho $a+b+c=0$ và $a b+b c+a c+4 m=0$ thì cũng tồn tại các số nguyên $a^{\prime}, b^{\prime}, c^{\prime}$ sao cho $a^{\prime}+b^{\prime}+c^{\prime}=0$ và $a^{\prime} b^{\prime}+b^{\prime} c^{\prime}+a^{\prime} c^{\prime}+m=0$.

(c) Với $k$ là số nguyên dương, chứng minh rằng không tồn tại các số nguyên $a, b, c$ khác 0 sao cho $a+b+c=0$ và $a b+b c+a c+2^{k}=0$.

Bài 3. Giả sử phương trình $2 x^{2}+2 a x+1-b=0$ có 2 nghiệm nguyên ( $a, b$ là tham số). Chứng minh rằng $a^{2}-b^{2}+2$ là số nguyên và không chia hết cho 3 .

Bài 4. Cho tam giác $A B C(A B<A C)$ có các góc nhọn, nội tiếp trong đường tròn tâm O. Gọi $M$ là trung điểm của cạnh $B C, E$ là điểm chính giữa của cung nhỏ $B C$, $F$ là điểm đối xứng của $E$ qua $M$.

(a) Chứng minh $E B^{2}=E F$.EO.

(b) Gọi $D$ là giao điểm của $A E$ và $B C$. Chứng minh các điểm $A, D, O, F$ cùng thuộc một đường tròn.

(c) Gọi $I$ là tâm đường tròn nội tiếp tam giác $A B C$ và $P$ là điểm thay đổi trên đường tròn ngoại tiếp tam giác $I B C$ sao cho $P, O, F$ không thẳng hàng. Chứng minh rằng tiếp tuyến tại $P$ của đường tròn ngoại tiếp tam giác POF đi qua một điểm cố định.

Bài 5. Để khuyến khích phong trào học tập, một trường THCS đã tổ chứng 8 đợt thi cho các học sinh. Ở mỗi đợt thi, có đúng 3 học sinh được chọn để trao giải. Sau khi tổ chứng xong 8 đợt thi, người ta nhận thấy rằng với hai đợt thi bât kì thì có đúng 1 học sinh được trao giải ở cả hai đợt thi đó. Chứng minh rằng:

(a) Có ít nhất một học sinh được trao giải ít nhất bốn lần.

(b) Có đúng một học sinh được trao giải ở 8 đợt thi.

LỜI GIẢI

 

Bài 1. (a) Giải phương trình $\sqrt{2 x-1}+\sqrt{1-2 x^{2}}=2 \sqrt{x-x^{2}}$.

(b) Cho các số $a$ và $b$ thỏa mãn điều kiện $\sqrt[3]{a}+\sqrt[3]{b}=\sqrt[3]{b-\frac{1}{4}}$. Chứng minh rằng $-1 \leq a<0$.

Lời giải.

(a) Đặt $a=\sqrt{2 x-1}, b=\sqrt{1-2 x^{2}}$.

Khi đó ta có $a+b=2 \sqrt{\frac{a^{2}+b^{2}}{2}} \Leftrightarrow a=b$.

Khi đó ta có $\sqrt{2 x-1}=\sqrt{1-2 x^{2}} \Leftrightarrow 2 x-1 \geq 0,2 x-1=1-2 x^{2}$.

Giải ra được nghiệm $x=\frac{-1+\sqrt{5}}{2}$.

(b) Ta có $x^{3}-y^{3}=(x-y)\left(x^{2}+x y+y^{2}\right)$ và $x^{2}+x y+y^{2} \geq 0$ nên $x \geq y \Leftrightarrow$ $x^{3} \geq y^{3}$.

Đặt $x=\sqrt[3]{a}, y=\sqrt[3]{b}$. Ta có $x+y=\sqrt[3]{y^{3}-\frac{1}{4}}$. Suy ra $x=\sqrt[3]{y^{3}-\frac{1}{4}}-y<0$.

Giả sử $x<-1$, ta có $\sqrt[3]{y^{3}-\frac{1}{4}}=y+x<y-1$

$\Leftrightarrow y^{3}-\frac{1}{4}<y^{3}-3 y^{2}+3 y-1$

$\Leftrightarrow y^{2}-y+\frac{1}{4}<0$ $\Leftrightarrow\left(y-\frac{1}{2}\right)^{2}<0$ (vô lý).

Do đó $x \geq-1 \Leftrightarrow a \geq-1$.

Vậy $-1 \leq a<0$.

Bài 2. (a) Tìm các số nguyên $a, b, c$ sao cho $a+b+c=0$ và $a b+b c+a c+$ $3=0$.

(b) Cho $m$ là số nguyên. Chứng minh rằng nếu tồn tại các số nguyên $a, b, c$ khác 0 sao cho $a+b+c=0$ và $a b+b c+a c+4 m=0$ thì cũng tồn tại các số nguyên $a^{\prime}, b^{\prime}, c^{\prime}$ sao cho $a^{\prime}+b^{\prime}+c^{\prime}=0$ và $a^{\prime} b^{\prime}+b^{\prime} c^{\prime}+a^{\prime} c^{\prime}+m=$ 0 .

(c) Với $k$ là số nguyên dương, chứng minh rằng không tồn tại các số nguyên $a, b, c$ khác 0 sao cho $a+b+c=0$ và $a b+b c+a c+2^{k}=0$.

Lời giải.

(a) Từ $a+b+c=0, a b+b c+c a=-3$ ta có $a^{2}+b^{2}+c^{2}=6$. Do $a, b, c$ vai trò như nhau nên ta có thể giả sử $|a| \geq|b| \geq|c|$. Khi đó $1<|a|<3$. Suy ra

$|a|=2$, suy ra $a=2$ hoặc $a=-2$.

Với $a=2$ thì $b+c=-2, b^{2}+c^{2}=2$ giải ra được $b=c=-1$.Ta có có bộ $(2 ;-1 ;-1)$ và các hoán vị.

Với $a=-2$ thì $b+c=2, b^{2}+c^{2}=2$, giải ra được $b=c=1$, ta có bộ $(-2 ; 1 ; 1)$ và hoán vị.

(b) Ta có $a+b+c=0$ chẵn (1)và $a b+b c+a c=-4 m$ chẵn.(2)

Nếu 3 số $a, b, c$ đều lẻ, không thỏa (1).

Nếu có 1 chẵn, 2 lẻ thì không thỏa (2).

Do đó 3 số $a, b, c$ đều chẵn. Khi đó đặt $a^{\prime}=\frac{a}{2}, b^{\prime}=\frac{b}{2}, c^{\prime}=\frac{c}{2}$ thì $a^{\prime}, b^{\prime}, c^{\prime}$ thỏa đề bài.

(c) Với $k=0$ ta có $a+b+c=0, a b+b c+a c=-1$ thì $a^{2}+b^{2}+c^{2}=2$ (3). Không có bộ 3 số nguyên $a, b, c$ khác 0 thỏa (3).

Với $k=1$ thì $a+b+c=0, a b+b c+a c=-2$ khi đó $a^{2}+b^{2}+c^{2}=4$ (4). Giả sử $|a|$ nhỏ nhất khi đó $1 \leq a^{2}<2$ (không có $a$ thỏa). Không tồn tại $a, b, c$ nguyên khác 0 thỏa (4).

Với $k>1$.

  • Nếu $k$ chẵn, đặt $k=2 n$ ta có $a+b+c=0, a b+b c+a c+4^{n}=0$, theo câu a), tồn tại $a_{1}, b_{1}, c_{1}$ nguyên thỏa

$a_{1}+b_{1}+c_{1}=0, a_{1} b_{1}+a_{1} c_{1}+b_{1} c_{1}+4^{n-1}=0$

Tương tự ta sẽ được $a_{n}, b_{n}, c_{n}$ nguyên thỏa $a_{n}+b_{n}+c_{n}=0, a_{n} b_{n}+$ $b_{n} c_{n}+a_{n} c_{n}=-1$ (vô nghiệm).

  • Nếu $k$ lẻ đặt $k=2 n+1$ ta có $a+b+c=0, a b+b c+a c+2.4^{n}=0$, làm tương tự trên ta được $a_{n}+b_{n}+c_{n}=0, a_{n} b_{n}+b_{n} c_{n}+a_{n} c_{n}=-2$ (vô nghiệm).

Vậy không tồn tại các số $a, b, c$ khác 0 thỏa đề bài.

Bài 3. Giả sử phương trình $2 x^{2}+2 a x+1-b=0$ có 2 nghiệm nguyên $(a, b$ là tham số). Chứng minh rằng $a^{2}-b^{2}+2$ là số nguyên và không chia hết cho 3 .

Lới giải.

Theo định lý Viete ta có $x_{1}+x_{2}=-a, x_{1} x_{2}=\frac{1-b}{2}$. Khi đó $Q=a^{2}-$ $b^{2}+2=\left(x_{1}+x_{2}\right)^{2}-\left(2 x_{1} x_{2}-1\right)^{2}+2=x_{1}^{2}+x_{2}^{2}-4 x_{1}^{2} x_{2}^{2}+6 x_{1} x_{2}+1$ là một số nguyên.

Ta chứng minh $Q$ không chia hết cho 3 .

Ta có tính chất sau, với một số nguyên $m$ bât kì thì nếu $m$ chia hết cho 3 thì $m^{2}$ chia hết cho 3 . Nếu $m$ chia 3 dư 1 hoặc 2 thì $m^{2}$ chia 3 dư 1 . Ta có $Q=x_{1}^{2}+x_{2}^{2}-x_{1}^{2} x_{2}^{2}+1-3 x_{1}^{2} x_{2}^{2}+6 x_{1} x_{2}$.

Ta cần chứng minh $Q^{\prime}=x_{1}^{2}+x_{2}^{2}-x_{1}^{2} x_{2}^{2}+1$ không chia hết cho 3 . Xét xác trường hợp sau:Nếu $x_{1}, x_{2}$ không chia hết cho 3 thì $x_{1}^{2}, x_{2}^{2}$ chia 3 dư 1 . Khi đó $Q^{\prime}$ chia 3 dư 2. Nếu $x_{1}$ chia hết cho $3, x_{2}$ không chia hết cho 3 , khi đó $Q^{\prime}$ chia 3 dư 2 .

$x_{1}, x_{2}$ chia hết cho 3 . Khi đó $Q^{\prime}$ chia 3 dư 1 .

Vậy $Q^{\prime}$ không chia hết cho 3 .

Do đó $Q$ không chia hết cho 3 .

Bài 4. Cho tam giác $A B C(A B<A C)$ có các góc nhọn, nội tiếp trong đường tròn tâm $O$. Gọi $M$ là trung điểm của cạnh $B C, E$ là điểm chính giữa của cung nhỏ $B C, F$ là điểm đối xứng của $E$ qua $M$.

(a) Chứng minh $E B^{2}=E F . EO$.

(b) Gọi $D$ là giao điểm của $A E$ và $B C$. Chứng minh các điểm $A, D, O, F$ cùng thuộc một đường tròn.

(c) Gọi $I$ là tâm đường tròn nội tiếp tam giác $A B C$ và $P$ là điểm thay đổi trên đường tròn ngoại tiếp tam giác $I B C$ sao cho $P, O, F$ không thẳng hàng. Chứng minh rằng tiếp tuyến tại $P$ của đường tròn ngoại tiếp tam giác POF đi qua một điểm cố định.

Lời giải.

(a) Ta có $E$ là điểm chính giữa cung $B C$, suy ra $E B=E C$ và $O E \perp B C$ nên $M, O, E$ thẳng hàng.

Vẽ đường kính $E K$. Ta có $E M \cdot E K=E B^{2}$.

Mặt khác $E F=2 E M, E O=\frac{1}{2} E K$. Do đó $E F \cdot E O=E M \cdot E K=E B^{2}$. (1)

(b) Ta có $\angle E B C=\angle E A C=\angle E A B$. Suy ra $\triangle E A B \sim \triangle E B D$. Suy ra $E B^{2}+$ $E D \cdot E A(2)$.

Từ (1) và (2) ta có: $E A \cdot E D=E O \cdot E F$. Suy ra tứ giác $O F D A$ nội tiếp.

(c) Ta có $\angle E I B=\angle E A B+\angle A B I=\frac{1}{2}(\angle A+\angle B)=\angle E B C+\angle C B I=\angle E B I$, suy ra $E B=E I=E C$. Vậy $E$ là tâm đường tròn ngoại tiếp tam giác $B I C$.

Do đó $E P=E B$. Ta có $E P^{2}=E B^{2}=E O \cdot E F$.

Suy ra $\triangle E P F \sim \angle E O P$. Suy ra $\angle E P F=\angle F O P$.

Hơn nữa, do $O, F$ cùng phía đối với $E$ nên $P O, P F$ cùng phía đối với $P E$.

Vẽ tia tiếp tuyến $P x(P F, P O$ cùng phía đối với $P x)$ của đường tròn ngoại tiếp tam giác $P O F$. Khi đó $\angle x P F=\angle F O P=\angle E P x$. Suy ra $P x$ và $P E$ trùng nhau. Vậy $P x$ luôn qua điểm $E$ cố định.

Bài 5. Để khuyến khích phong trào học tập, một trường THCS đã tổ chứng 8 đợt thi cho các học sinh. Ở mỗi đợt thi, có đúng 3 học sinh được chọn để trao giải. Sau khi tổ chứng xong 8 đợt thi, người ta nhận thấy rằng với hai đợt thi bất kì thì có đúng 1 học sinh được trao giải ở cả hai đợt thi đó. Chứng minh rằng:

(a) Có ít nhất một học sinh được trao giải ít nhất bốn lần.

(b) Có đúng một học sinh được trao giải ở 8 đợt thi.

Lời giải.

(a) Giả sử $A_{1}$ là tập 3 bạn đạt giải trong đợt thi thứ nhât. Tương tự với $A_{2}, \ldots, A_{8}$.

Ta có $A_{1}={a, b, c}$. Vị $A_{1} \cap A_{i}, i=\overline{2,8}$ có đúng một học sinh nên các học sinh $a, b, c$ xuất hiện trong 7 tập $A_{2}, \ldots, A_{8}$ và không có hai bàn nào xuất hiện cùng một tập. Do đó theo nguyên lí Đirichlet thì có 1 học sinh thuộc ít nhất 3 tập trong các tập $A_{2}, \ldots, A_{8}$. Khi đó học sinh này có xuất hiện trong ít nhất 4 tập, hay được nhận thưởng ít nhất 4 lần.

(b) Theo câu a, có một học sinh $a$ nhận thưởng được ít nhất 4 lần, giả sử là từ lần 1 đến lần 4 . Hay $a$ thuộc $A_{1}, A_{2}, A_{3}, A_{4}$. Khi đó nếu $a$ không nhận thưởng trong 8 lần, tức là có một lần $a$ không nhận thưởng. Giả sử là lần 8 , tức là $a$ không thuộc $A_{8}$.

Khi đó $A_{1} \cap A_{8}$ là 1 học sinh nên có học $\sinh b \neq a$ thuộc $A_{8}$, tương tự có học sinh $c, d, e$ lần lượt thuộc $A_{2}, A_{3}, A_{4}$ cũng thuộc $A_{8}$. Hơn nữa $b, c, d, e$ phải phân biệt. Do đó $A_{8}$ chứa ít nhất 4 phần tử. (vô lý). Vậy có một học sinh thuộc 8 tập, hay nhận thưởng 8 lần. Và không có hai học sinh nào cùng nhận thưởng hai lần nên chỉ có đúng một học sinh thỏa.

 

 

 

 

 

 

 

 

 

 

 

 

 

 

 

 

Đề thi và đáp án kì thi chọn đội tuyển thi Quốc gia trường Phổ thông Năng khiếu năm học 2015 -2016

Đề bài

Ngày thi thứ nhất

Bài 1.  Cho tập hợp:
$$A=\left\{ n\in \mathbb{N}\mid 1\le n\le 2015,\gcd (n,2016)=1 \right\}.$$
Hỏi có bao nhiêu số nguyên $a\in A$ sao cho tồn tại số nguyên $b$ mà $a+2016b$ là số chính phương?
Bài 2. Cho $a,b,c,d$ là các số thực thỏa mãn điều kiện:
$$\left\{\begin{array}{l}
a^2\le 1\\
a^2+b^2\le 5\\
a^2+b^2+c^2\le 14\\
a^2+b^2+c^2+d^2\le 30
\end{array} \right..$$

a)Chứng minh rằng $a+b+c+d\le 10$.
b) Chứng minh rằng $ad+bc\le 10$.

Bài 3.  Tìm tất cả các hàm số $f:\mathbb{R}\to \mathbb{R}$ thỏa mãn:
$$f\left( x-2f(y) \right)=5f(x)-4x-2f(y), \, \ \forall x,y\in \mathbb R.$$
Bài 4. Cho đường tròn $k$ và các điểm $B,C$ thuộc đường tròn sao cho $BC$ không phải là đường kính; $I$ là trung điểm $BC$. Điểm $A$ di động trên cung lớn $BC$ của $k$. Gọi $(\mathcal I_1)$ là đường tròn qua $I$ và tiếp xúc với $AB$ tại $B$, $(\mathcal I_2)$ là đường tròn qua $I$ và tiếp xúc với $AC$ tại $C$. Các đường tròn $(\mathcal I_1), (\mathcal I_2)$ cắt nhau tại $D$.

a) Chứng minh rằng đường tròn ngoại tiếp tam giác $AID$ luôn đi qua một điểm cố định.
b) Gọi $K$ là trung điểm $AD$, $E$ là tâm đường tròn qua $K$ và tiếp xúc với $AB$ tại $A$, $F$ là tâm đường tròn qua $K$ và tiếp xúc với $AC$ tại $A$. Chứng minh rằng góc $\angle EAF$ có số đo không đổi.

Ngày thi thứ hai

Bài 5. Cho dãy số $(x_n)$ được xác định bởi $x_n=\dfrac{1}{n\cos \frac{1}{n}}\ \forall n\in \mathbb N^*$. Tính giới hạn:
$$\lim_{n \to +\infty} \frac{{{x}_{1}}+{{x}_{3}}+{{x}_{5}}+\cdots+{{x}_{2n-1}}}{{{x}_{2}}+{{x}_{4}}+{{x}_{6}}+\cdots +{{x}_{2n}}}.$$
Bài 6. Tìm các giá trị của $b$ sao cho tồn tại $a$ để hệ phương trình sau có nghiệm $(x,y)$:
$$\left\{\begin{array}{l}
(x-1)^2+(y+1)^2=b \\
y=x^2+(2a+1)x+a^2
\end{array} \right..$$
Bài 7. Cho $n$ là số nguyên dương, $n\ge 2$ và $X=\left\{ 1,2,3,\ldots,n \right\}$. Gọi ${{A}_{1}},{{A}_{2}},\ldots,{{A}_{m}}$ và ${{B}_{1}},{{B}_{2}},\ldots,{{B}_{m}}$ là hai dãy các tập con khác rỗng của $X$ thỏa mãn điều kiện sau: với mỗi $i,j\in \left\{ 1,2,3,\ldots,m \right\}$, ${{A}_{i}}\cap {{B}_{j}}=\varnothing $ nếu và chỉ nếu $i=j.$
a) Chứng minh rằng với mỗi hoán vị $(x_1,x_2,\ldots,x_n)$ của tập hợp $X$, có không quá một cặp tập hợp $(A_i,B_i)$ với $i=1,2,3,\ldots,m$ sao cho nếu $x_k\in A_i$ và $x_l\in B_i$ thì ta phải có $k<l$.
b) Gọi $a_i,b_i$ lần lượt là số phần tử của tập hợp $A_i,B_i$ với $i=1,2,3,\ldots,m$. Chứng minh rằng:
$$\sum_{i=1}^{m}{\dfrac{1}{C_{{a_i+b_i}}^{a_i}}}\le 1.$$

Bài 8.  Cho tam giác $ABC$ nhọn nội tiếp đường tròn tâm $O$. Đường tròn tâm $I$ đi qua $B,C$ lần lượt cắt các tia $BA,CA$ tại $E,F.$

a) Giả sử các tia $BF,CE$ cắt nhau tại $D$ và $T$ là tâm đường tròn $(AEF)$. Chứng minh rằng $OT\parallel ID.$
b) Trên $BF,CE$ lần lượt lấy các điểm $G,H$ sao cho $AG\perp CE,AH\perp BF.$ Các đường tròn $(ABF),(ACE)$ cắt $BC$ tại $M,N$ khác $B,C$ và cắt $EF$ tại $P,Q$ khác $E,F$. Gọi $K$ là giao điểm của $MP,NQ$. Chứng minh rằng $DK\perp GH$.

Hết

Giải

Bài 1.

Cho số nguyên dương $n>1$, ta quy ước gọi một số nguyên dương $a < n$ là thặng dư chính phương theo modulo $n$ nếu $\gcd(a,n)=1$ và tồn tại số nguyên $x$ sao cho $a\equiv {{x}^{2}} \pmod n .$ \medskip

Đặt $s(n)$ là số các số như thế. Ta sẽ chứng minh hai bổ đề dưới đây: \medskip

Bổ đề 1. Cho $p$ là số nguyên tố và $k$ là số nguyên dương. Khi đó:

Nếu $p=2$ thì $s({{2}^{k}})={{2}^{\max (k-3,0)}}$.
Nếu $p>2$ thì $s({{p}^{k}})=\dfrac{{{p}^{k}}-{{p}^{k-1}}}{2}$.

Bổ đề 2. $s(n)$ là hàm nhân tính, tức là $s(ab)=s(a)s(b)$ với $\gcd(a,b)=1$. \medskip

Thật vậy, \medskip

Trước hết, ta biết rằng $s(p)=\frac{p-1}{2}$ với $p$ là số nguyên tố lẻ. Ta sẽ tính $s({{p}^{k}})$ với $k\in {{\mathbb{Z}}^{+}}$. Xét một thặng dư chính phương $a$ của $p$, khi đó tồn tại $x$ sao cho $$a\equiv {{x}^{2}}\pmod{p}.$$ Đặt $a={{x}^{2}}+pq$ thì hiển nhiên $$a\equiv {{x}^{2}}+pq \pmod {{p}^{k}}\Leftrightarrow a-pq\equiv {{x}^{2}} \pmod {{p}^{k}}$$ và khi đó, ta có ${{p}^{k-1}}$ cách chọn $q$ để các số $a-pq$ là các thặng dư chính phương theo modulo ${{p}^{k}}$. Suy ra
$$s({{p}^{k}})={{p}^{k-1}}s(p)=\frac{{{p}^{k}}-{{p}^{k-1}}}{2}.$$ Xét số nguyên tố $p=2$, với $k=1,2,3,$ dễ dàng kiểm tra được $s({{2}^{k}})=1$. \medskip

Ta xét $k\ge 4$, tương tự trên, ở bước chọn $q$, ta chỉ có 2 cách nên $s({{2}^{k}})=2s({{2}^{k-1}})$. Từ đó bằng quy nạp, ta có được $$s({{2}^{k}})={{2}^{k-3}},k\ge 4.$$ Tiếp theo, xét hai số $a,b$ nguyên dương nguyên tố cùng nhau. Gọi $A$ là tập hợp các thặng dư chính phương theo modulo $ab$ và $B$ là tập hợp các số là thặng dư chính phương chung của $a,b.$ \medskip

Nếu $x\in A$ thì tồn tại $y$ sao cho $x\equiv {{y}^{2}} \pmod{ab}$. Rõ ràng khi đó,
$$x\equiv {{y}^{2}}\pmod a, \, x\equiv {{y}^{2}}\pmod b$$
(chú ý rằng nếu $x>a$, ta có thể chọn ${x}'$ sao cho ${x}'<a$ và $x\equiv {x}'\pmod a$; tương tự với $b$).
Do đó, $x\in B$, tức là $x\in A\Rightarrow x\in B$ nên $\left| A \right|\le \left| B \right|$. \medskip

Tiếp theo, xét $x\in B$. Khi đó tồn tại $r,s$ sao cho
$x\equiv {{r}^{2}}\pmod a,\text{ }x\equiv {{s}^{2}}\pmod b$.
Theo định lý thặng dư Trung Hoa, tồn tại số nguyên $z$ sao cho $$z\equiv r\pmod a, \, z\equiv s\pmod b.$$ Khi đó $$x\equiv {{z}^{2}}\pmod a, \, x\equiv {{z}^{2}}\pmod b$$ nên
$$x\equiv {{z}^{2}} \pmod{ab}.$$ Do đó: $x\in A$, tức là $x\in B\Rightarrow x\in A$ nên $\left| A \right|\ge \left| B \right|$. Từ đây ta có $$\left| A \right|=\left| B \right| \text{ hay } s(a)s(b)=s(ab).$$ Vậy $s(n)$ là hàm nhân tính. \medskip

Các bổ đề đều được chứng minh. \medskip

Trở lại bài toán, ta thấy rằng $2016={{2}^{5}}\cdot {{3}^{2}}\cdot 7.$ Rõ ràng bài toán yêu cầu đếm số thặng dư chính phương theo modulo $2016$.
Theo bổ đề 2 thì $$s(2016)=s({{2}^{5}})s({{3}^{2}})s(7).$$ Theo bổ đề 1 thì $$s({{2}^{5}})={{2}^{2}}=4,s({{3}^{2}})=\frac{{{3}^{2}}-3}{2}=3,s(7)=\frac{7-1}{2}=3.$$ Do đó, số các số $a$ cần tìm là $4\cdot 3\cdot 3=36.$

Bài 2. 

(a) Dự đoán dấu bằng xảy ra khi $a=1,b=2,c=3,d=4$ nên ta có các đánh giá sau $
{{a}^{2}}+1\ge 2a
{{b}^{2}}+4\ge 4b
{{c}^{2}}+9\ge 6c
{{d}^{2}}+16\ge 8d
$

Do đó, ta có
$24(a+b+c+d)\le 3({{d}^{2}}+16)+4({{c}^{2}}+9)+6({{b}^{2}}+4)+12({{a}^{2}}+1)$
$=3{{d}^{2}}+4{{c}^{2}}+6{{b}^{2}}+12{{a}^{2}}+120 $

$=3({{a}^{2}}+{{b}^{2}}+{{c}^{2}}+{{d}^{2}})+({{a}^{2}}+{{b}^{2}}+{{c}^{2}})+2({{a}^{2}}+{{b}^{2}})+6{{a}^{2}}+120$

$\le 3\cdot 30+14+2\cdot 5+6\cdot 1+120=240$
Suy ra $a+b+c+d\le 10.$ \medskip

(b) Ta có $$16{{a}^{2}}+{{d}^{2}}\ge 8ad \text{ và } 9{{b}^{2}}+4{{c}^{2}}\ge 12bc.$$

Từ đó suy ra

$24(ad+bc)\le 3(16{{a}^{2}}+{{d}^{2}})+2(9{{b}^{2}}+4{{c}^{2}})$
$=3({{a}^{2}}+{{b}^{2}}+{{c}^{2}}+{{d}^{2}})+5({{a}^{2}}+{{b}^{2}}+{{c}^{2}})+10({{a}^{2}}+{{b}^{2}})+30{{a}^{2}}$
$\le 3\cdot 30+5\cdot 14+10\cdot 5+30\cdot 1=240$
Suy ra $ad+bc\le 10.$

Bài 3.

Gọi () là điều kiện đề bài cho.
Trong (
) thay $x=y=0$, ta có $$f(-2f(0))=3f(0).$$ Đặt $f(0)=a$ thì $f(-2a)=3a$.
Trong () thay $x=0$ và $y=-2a$, ta có $$f(-2f(-2a))=5a-2f(-2a)\Leftrightarrow f(-6a)=-a.$$ Trong (), thay $x=-2a,y=-6a$, ta có
$$\begin{aligned}
& f(-2a-2f(-6a))=5f(-2a)-4x-2f(-6a) \\
& \Leftrightarrow f(0)=15a+8a+2a.
\end{aligned}$$ Từ đây ta có $a=25a$ nên $a=0,$ tức là $f(0)=0$. \medskip

Trong $(*),$ thay $y=0$, ta có $$f(x)=5f(x)-4x\Leftrightarrow f(x)=x.$$ Thử lại ta thấy thỏa. Vậy hàm số cần tìm chính là $f(x)=x,\forall x\in \mathbb{R}.$

Bài 4.

(a) Gọi $O$ là tâm của đường tròn $k.$ Không mất tính tổng quát, giả sử tia $AD$ nằm giữa hai tia $AO,AB,$ các trường hợp còn lại tương tự.

Ta có: $$\angle IDB=\angle ABC,\angle IDC=\angle ACB$$ nên $$\angle BAC+\angle BDC=\angle BAC+\angle ABC+\angle ACB=180{}^\circ .$$ Do đó, tứ giác $ABDC$ nội tiếp hay $D\in (O).$ Ta thấy $$\begin{aligned}
& \angle DAO+\angle OID \\
& =\angle BAC-(\angle DAB+\angle OAC)+360{}^\circ -(90{}^\circ +\angle DIC) \\
& =\angle BAC-\left( \angle ICD+90{}^\circ -\angle ABC \right)+270{}^\circ -\angle DIC \\
& =\angle BAC+\angle ABC-(\angle ICD+\angle DIC)+180{}^\circ \\
& =(180{}^\circ -\angle ACB)-\left( 180{}^\circ -\angle IDC \right)+180{}^\circ \\
& =\angle IDC-\angle ACB+180{}^\circ =180{}^\circ.
\end{aligned} $$

Do đó, $AOID$ nội tiếp hay đường tròn $(AID)$ đi qua $O$ cố định. \medskip

(b) Ta có: $$\angle EAC=90{}^\circ -\angle BAC,\angle FAB=90{}^\circ -\angle BAC$$ nên
$$\angle EAF=180{}^\circ -2\angle BAC+\angle BAC=180{}^\circ -\angle BAC.$$

Do đó, góc $\angle EAF$ có số đo không đổi.

Bài 5.

Trước hết, ta chứng minh bổ đề sau: \medskip

Bổ đề. Giá trị của biểu thức $\frac{1}{1}+\frac{1}{2}+\frac{1}{3}+\cdots+\frac{1}{n}$ tiến tới vô cực khi $n\to +\infty.$ \medskip

Thật vậy, xét hàm số $f(x)=\ln (1+x)-x$ với $x>0$. Ta có $$f'(x)=\frac{1}{1+x}-1<0$$ nên đây là hàm nghịch biến, suy ra $f(x)<f(0)=0$ hay $\ln (1+x)<x,\forall x>0$. Thay $x$ bởi $\frac{1}{n}$, ta được
$$\ln \left( 1+\frac{1}{n} \right)<\frac{1}{n}\Leftrightarrow \frac{1}{n}>\ln (1+n)-\ln n.$$ Do đó, $$\frac{1}{1}+\frac{1}{2}+\frac{1}{3}++\frac{1}{n}>\ln 2-\ln1+\ln3-\ln2+\cdots+ln(n+1)-\ln n=\ln (n+1).$$ Vì $\ln (n+1)\to +\infty $ khi $n\to +\infty $ nên $$\frac{1}{1}+\frac{1}{2}+\frac{1}{3}+\cdots+\frac{1}{n}\to +\infty.$$
Trở lại bài toán, đặt $$y_n=\frac{x_1+x_3+x_5+\cdots+x_{2n-1}}{x_2+x_4+x_6+\cdots+x_{2n}}$$ với $n\ge 1.$

Ta thấy vì $\frac{1}{n}\in \left( 0;\frac{\pi }{2} \right)$ nên $\cos \frac{1}{n}>0$, suy ra $$x_n=\frac{1}{n\cos \frac{1}{n}}>0,n\ge 1. $$
Xét hàm số $f(t)=\frac{t}{\cos t}$ với $t\in \left( 0;\frac{\pi }{2} \right)$ thì ${f}'(t)=\frac{\cos t+t\sin t}{{{\cos }^{2}}t}>0$ nên đây là hàm đồng biến. Chú ý rằng $x_n=f\left( \frac{1}{n} \right)$, mà $\frac{1}{n}$ là dãy giảm nên $x_n$ cũng là dãy giảm. \medskip

Suy ra $x_1>x_2,x_3>x_4,\ldots,x_{2n-1}>x_{2n}$ nên $y_n>1$. \medskip

Ngoài ra, ta cũng có $x_3<x_2,x_5<x_4,\ldots,x_{2n-1}<x_{2n-2}$ nên $y_n< \frac{x_1+x_2+x_4+\cdots+x_{2n-2}}{x_2+x_4+\cdots+x_{2n}}$

$= 1-\frac{x_1-x_{2n}}{x_2+x_4+\cdots+x_{2n}}<1-\frac{x_1}{x_2+x_4+\cdots+x_{2n}}$

Dễ thấy rằng $$x_2+x_4+\cdots+x_{2n}=\sum\limits_{i=1}^{n}{\frac{1}{2i\cos \frac{1}{2i}}}\ge \sum\limits_{i=1}^{n}{\frac{1}{2i}}=\frac{1}{2}\sum\limits_{i=1}^{n}{\frac{1}{i}}.$$

Theo bổ đề trên thì $\sum\limits_{i=1}^{n}{\frac{1}{i}}$ tiến tới vô cực nên $$\lim \left( x_2+x_4+\cdots+x_{2n} \right)=+\infty .$$

Do đó $$\lim \left( 1-\frac{x_{1}}{x_2+x_4+\cdots+x_{2n}} \right)=1-0=1.$$ Theo nguyên lý kẹp, ta có $\lim x_n=1.$

Bài 6.

Đặt $X=x-1,Y=y+1$, thay vào, ta có
$$\begin{aligned}
& \left\{ \begin{aligned}
& {{X}^{2}}+{{Y}^{2}}=b \\
& Y-1={{(X+1)}^{2}}+(2a+1)(X+1)+{{a}^{2}} \\
\end{aligned} \right. \
& \Leftrightarrow \left\{ \begin{aligned}
& {{X}^{2}}+{{Y}^{2}}=b \\
& Y={{X}^{2}}+(2a+3)X+{{a}^{2}}+2a+3.
\end{aligned} \right. \\
\end{aligned}$$
Ta đưa về tìm điều kiện của $b$ để tồn tại $a$ mà hệ trên có nghiệm $(X,Y).$ Do $$Y-(X+2)={{X}^{2}}+2(a+1)X+{{(a+1)}^{2}}={{\left( X+a+1 \right)}^{2}}\ge 0$$ nên $Y\ge X+2$. Suy ra $Y-X\ge 2>0$, tức là ${{(X-Y)}^{2}}\ge 4.$ Ta có
$$b={{X}^{2}}+{{Y}^{2}}=\frac{{{(X-Y)}^{2}}+{{(X+Y)}^{2}}}{2}\ge \frac{{{(Y-X)}^{2}}}{2}\ge 2.$$ Mặt khác, với $b\ge 2$, nếu chọn $X=-(a+1)$ thì có $Y=X+2=1-a$. Khi đó, ta có
$${{X}^{2}}+{{Y}^{2}}={{(a+1)}^{2}}+{{(a-1)}^{2}}=2({{a}^{2}}+1)=b.$$ Như thế, với $a$ thỏa mãn $2({{a}^{2}}+1)=b$ thì hệ có nghiệm là $$(X,Y)=(-a-1,1-a).$$ Dễ dàng thấy rằng do $b\ge 2$ nên luôn tồn tại $a$ như thế. \medskip

Vậy các giá trị cần tìm của $b$ là $b\ge 2$.

Bài 7.

(a) Giả sử ngược lại, tồn tại $2$ cặp $(A_i,B_i)$ và $(A_j,B_j)$ thỏa mãn điều kiện đề bài đã cho. \medskip

Vì $i\ne j$ nên theo giả thiết, $$\left| A_i \cap B_j \right|\ge 1,\left| A_j\cap B_i \right|\ge 1.$$ Đặt $x_r\in A_i\cap B_j,x_s\in A_j\cap B_i$ với $1\le r,s\le n$ thì:

Do $x_r\in B_j$ nên với mọi $x_k\in A_j$, ta đều có $k<r.$
Do $x_r\in A_i$ nên với mọi $x_k\in B_i$, ta đều có $k>r$.

Từ đây suy ra $$A_j \subset \left\{ x_1,x_2,\ldots,x_{r-1} \right\},B_i\subset \left\{x_{r+1},x_{r+2},\ldots,x_n \right\}.$$

Điều này cho thấy $A_j\cap B_i=\varnothing $, mâu thuẫn với giả thiết. Vậy tồn tại không quá $1$ cặp $(A_i,B_i)$ thỏa mãn điều kiện đã cho. \medskip

(b) Gọi $T$ là tập hợp các cách chọn hai dãy $$A_1,A_2,\ldots,A_m \text{và} B_1,B_2,\ldots,B_m$$ thỏa mãn điều kiện là: với mỗi $i,j\in \left\{ 1,2,3,\ldots,n \right\}$, $A_i\cap B_j=\varnothing $ nếu và chỉ nếu $i=j.$ \medskip

Gọi $T_i\subset T$ là các cách chọn sao cho sao cho cặp $(A_i,B_i)$ thỏa mãn điều kiện là: cặp $(A_i,B_i)$ với $i=1,2,3,\ldots,n$ sao cho nếu $x_k \in A_i$ và $x_l\in B_i$ thì $x_k<x_l$ (ở đây ta xét thứ tự ban đầu của các phần tử của $X$). \hfill (*) \medskip

Theo câu (a) thì $T_i \cap T_j=\varnothing $ với $i\ne j$ nên ta có $$\left| T_1 \right|+\left| T_2 \right|+\cdots +\left| T_m \right|=\left| T_1 \cup T_2 \cup \ldots \cup T_m \right|\le T.$$ Tiếp theo, với $1\le i\le m$, xét một tập hợp $S\subset X$ và $\left| S \right|=a_i+b_i$. Khi đó, tương ứng với $S$, có đúng $1$ cách chọn $(A_i,B_i)$ thỏa mãn tính chất $(*)$ – tức là $A_i$ sẽ nhận $a_i$ số nhỏ nhất trong tập $S,$ $B_i$ là lấy phần còn lại. \medskip

Trong khi đó, nếu không có điều kiện $(*),$ ta có thể chọn tùy ý $C_{a_i+b_i}^{a_i}$ phần tử trong $S$ và $A$ và số còn lại cho $B.$ \medskip

Do đó, ta có
$\left| T_i \right|=\frac{\left| T \right|}{C_{a_i+b_i}^{a_i}} $ với $i=1,2,\ldots,m.$

Từ đây suy ra

$$\sum\limits_{i=1}^{m}\frac{\left| T \right|}{C_{a_i+b_i}^{a_i}}\le \left| T \right|\Leftrightarrow \sum\limits_{i=1}^{m}\frac{1}{C_{a_i+b_i}^{a_i}}\le 1$$

Ta có đpcm.

Bài 8. 

(a) Giả sử $EF$ cắt $BC$ ở $L$ và $(T),(O)$ cắt nhau tại $J$ khác $A.$ Suy ra $AJ$ chính là trục đẳng phương của $(T),(O).$ Do đó $OT\bot AJ$. \medskip

Khi đó,
[LB\cdot LC=LE\cdot LF] nên $L$ thuộc trục đẳng phương của $(T),(O)$. Suy ra $A,J,L$ thẳng hàng. Theo định lý Brocard cho tứ giác $BEFC$ nội tiếp trong đường tròn $(I)$ thì $I$ chính là trực tâm của tam giác $ADL.$ \medskip

Vì thế nên $ID\bot AL$, mà $OT\bot AJ$ nên $ID\parallel OT$. \medskip

(b) Dễ dàng thấy rằng $D$ là trực tâm của tam giác $AGH$ nên $AD\bot GH$. Ta sẽ chứng minh rằng $A,D,K$ thẳng hàng. \medskip

Ta có $DB\cdot DF=DE\cdot DC$ nên $D$ có cùng phương tích tới $(ABF),(AEC)$. Suy ra $AD$ chính là trục đẳng phương của $2$ đường tròn này. \medskip

Bằng biến đổi các góc nội tiếp, ta thấy rằng
$$\angle MPQ=\angle MBF=\angle CEF=\angle CNQ.$$ Suy ra $MNPQ$ nội tiếp, dẫn đến $KM\cdot KP=KN\cdot KQ$, tức là $K$ cũng có cùng phương tích tới $2$ đường tròn $(ABF),(AEC)$. \medskip

Từ đó suy ra $A,D,K$ thẳng hàng. Do đó, $DK$ vuông góc với $GH.$

Đáp án thi chọn Đội Tuyển thi Quốc Gia của trường PTNK năm học 2015 – 2016

Ngày thứ 1

Bài 1. Cho tập hợp
$$
A=\{n \in \mathbb{N} \mid 1 \leq n \leq 2015,(n, 2016)=1\}
$$
Hỏi có bao nhiêu số nguyên $a \in A$ sao cho tồn tại số nguyên b mà $a+2016 b$ là số chính phương?

Bài 2. Cho $a, b, c, d$ là các số thực thỏa mãn điều kiện
$$
a^{2} \leq 1, a^{2}+b^{2} \leq 5, a^{2}+b^{2}+c^{2} \leq 14, a^{2}+b^{2}+c^{2}+d^{2} \leq 30
$$
1. Chúng minh rằng $a+b+c+d \leq 10$.
2. Chứng minh rằng $a d+b c \leq 10$.

Bài 3. Tìm tất cả các hàm số $f: \mathbb{R} \rightarrow \mathbb{R}$ thỏa mãn điều kiện
$$
f(x-2 f(y))=5 f(x)-4 x-2 f(y)
$$
với mọi $x, y \in \mathbb{R}$.

Bài 4. Cho đường tròn $k$ và các điểm $B, C$ thuộc đường tròn, không phải là đường kính; I là trung điểm $B C$. Điểm $A$ di động trên cung lớn $B C$ của $k$. Gọi $i_{1}$ là đường tròn qua $I$ và tiếp xúc với $A B$ tại $B ; i_{2}$ là đường tròn qua $I$ và tiếp xúc với $A C$ tại $C$. Các đường tròn $i_{1}, i_{2}$ cắt nhau tại $D$ (khác $I$ ).
1. Chứng minh rằng đường tròn ngoại tiếp tam giác AID luôn đi qua một điểm cố định.
2. Gọi $K$ là trung điểm $A D$, $E$ là tâm đường tròn qua $K$ và tiếp xúc với $A B$ tại $A, F$ là tâm đường tròn qua $K$ và tiếp xúc với AC tại $A$. Chứng minh rằng góc EAF có số đo không đổi.

Ngày thứ 2

Bài 5. Dãy số $\left(x_{n}\right)$ được xác định bởi công thức $x_{n}=\frac{1}{n \cos \frac{1}{n}}$ với mọi $n \geq 1$. Tính giới hạn sau
$$\lim \frac{x_{1}+x_{3}+x_{5}+\cdots+x_{2 n-1}}{x_{2}+x_{4}+x_{6}+s+x_{2 n}}$$

Bài 6. Tim các giá trị của $b$ sao cho tồn tại a để hệ phương trình sau có nghiệm $(x, y)$
$$
\left\{\begin{array}{l}
(x-1)^{2}+(y+1)^{2}=b \\y=x^{2}+(2 a+1) x+a^{2}
\end{array}\right.
$$

Bài 7. Cho n là số nguyên dương, $n \geq 2$ và $X={1,2,3, \ldots, n}$. Gọi $A_{1}, A_{2}, \ldots, A_{m}$ và $B_{1}, B_{2}, \ldots, B_{m}$ là hai dãy các tập con khác rỗng của $X$ thỏa mãn điều kiện: Với mỗi $i, j \in{1,2,3, \ldots, n}, A_{i} \cap B_{j}=\varnothing$ nếu và chỉ nếu $i=j$.
1. Chúng minh rằng với mỗi hoán vị $\left(x_{1}, x_{2}, \ldots, x_{n}\right)$ của $X$, có không quá một cặp tập hợp $\left(A_{i}, B_{i}\right)$ với $i=1,2,3, \ldots, n$ sao cho nếu $x_{k} \in A_{i}$ và $x_{l} \in B_{i}$ thì $k<l$.
2. Gọi $a_{i}, b_{i}$ lần lượt là số phần tử của tập hợp $A_{i}, B_{i}$ với $i=1,2,3, \ldots, m$. Chúng minh rằng
$$
\sum_{i=1}^{m} \frac{1}{C_{a_{i}+b_{i}}^{a_{i}}} \leq 1
$$

Bài 8. Cho tam giác $A B C$ nhọn nội tiếp đường tròn tâm $O$. Đường tròn tâm $I$ đi qua $B$, $C$ lần lượt cắt các tia $B A$, CA tại $E, F$.
1. Giả sử các tia $B F, C E$ cắt nhau tại $D$ và $T$ là tâm đường tròn $(A E F)$. Chứng minh rằng $O T$ || ID.
2. Trên BF, CE lần lượt lấy các điểm $G, H$ sao cho $A G \perp C E, A H \perp B F$. Các đường tròn $(A B F),(A C E)$ cắt $B C$ tai $M, N$ (khác $B, C)$ và cắt EF tại $P, Q$ (khác $E, F)$. Gọi $K$ là giao điểm của $M P, N Q$. Chứng minh rằng DK vuông góc với GH.

Giải

Bài 1.

Cho $n$ là số nguyên dương lớn hơn 1 , ta quy ước gọi một số nguyên dương a được gọi là thặng dư chính phương theo modulo $n$ nếu $(a, n)=1$ và tồn tại số nguyên $x$ sao cho $a \equiv x^{2}(\bmod n)$. Trong bài này, dể dơn giản, ta quy ước xét các thặng dư chính phưong nhỏ hơn $n$.
Đặt $s(n)$ là số các số nhỏ hơn $n$ và là thặng dư chính phương theo modulo n. Ta sễ chứng minh hai bổ dề dưới đây:
Bổ đề 1: Cho $p$ là số nguyên tố và $k$ là số nguyên dưong. Khi đó:
1. Nếu $p=2$ thì $s\left(2^{k}\right)=2^{\max (k-3,0)}$.
2. Nếu $p>2$ thì $s\left(p^{k}\right)=\frac{p^{k}-p^{k-1}}{2}$.
Bổ đề $2: s(n)$ là hàm nhân tính.
Thật vậy,
Trước hết, ta biết rằng $s(p)=\frac{p-1}{2}$ với $p$ là số nguyên tố lẻ. Ta sẽ tính $s\left(p^{k}\right)$ với $k \in \mathbb{Z}^{+}$.
Xét một thặng dư chính phương $a$ của $p$, khi đó tồn tại $x$ sao cho
$$
a \equiv x^{2}(\bmod p)
$$
Dặt $a=x^{2}+p q$ thì hiển nhiên
$$
a \equiv x^{2}+p q\left(\bmod p^{k}\right) \Leftrightarrow a-p q \equiv x^{2}\left(\bmod p^{k}\right)
$$
và khi đó, ta có $p^{k-1}$ cách chọn $q$ để các số $a-p q$ là các thặng dư chính phưong $\bmod p^{k}$.
Suy ra
$$
s\left(p^{k}\right)=p^{k-1} s(p)=\frac{p^{k}-p^{k-1}}{2}
$$
Xét số nguyên tố $p=2$, với $k=1,2,3$, dể dàng kiểm tra được $s\left(2^{k}\right)=1$.
Ta xét $k \geq 4$, tưong tự trên, ờ bước chọn $q$, ta chỉ có 2 cách nên $s\left(2^{k}\right)=$ $2 s\left(2^{k-1}\right)$. Từ đó bằng quy nạp, ta có được
$$
s\left(2^{k}\right)=2^{k-3}, k \geq 4
$$
Tiếp theo, xét hai số $a, b$ nguyên dương và $(a, b)=1 .$ Gọi $A$ là tập họp các thặng dư chính phương theo modulo $a b$ và $B$ là tập hợp các số là thặng dư chính phưong chung của $a, b$.
Nếu $x \in A$ thì tổn tại $y$ sao cho $x \equiv y^{2}(\bmod a b)$. Rō ràng khi đó,
$$
x \equiv y^{2} \quad(\bmod a), x \equiv y^{2} \quad(\bmod b)
$$

(chú ý rằng nếu $x>a$, ta có thể chọn $x^{\prime}$ sao cho $x^{\prime}<a$ và $x \equiv x^{\prime}(\bmod a)$; tương tự với $b$ ). Do đó, $x \in B$, tức là $x \in A \Rightarrow x \in B$ nên $|A| \leq|B|$.
Tiếp theo, xét $x \in B$. Khi đó tồn tại $r, s$ sao cho $x \equiv r^{2}(\bmod a), x \equiv s^{2}$ $(\bmod b)$. Theo định lý thặng dư Trung Hoa, tổn tại số nguyên $z$ sao cho
$$
z \equiv r(\bmod a), z \equiv s(\bmod b)
$$
Khi đó
$$
x \equiv z^{2} \quad(\bmod a), x \equiv z^{2} \quad(\bmod b)
$$
nên
$$
x-z^{2}: a b \text { hay } x \equiv z^{2}(\bmod a b)
$$
Do đó: $x \in A$, tức là $x \in B \Rightarrow x \in A$ nên $|A| \geq|B|$.
Từ đây ta có
$$
|A|=|B| \text { hay } s(a) s(b)=s(a b)
$$
Vậy $s(n)$ là hàm nhân tính.
Các bổ đề đều được chứng minh.
Trở lại bài toán, ta thấy rằng
$$
2016=2^{5} \cdot 3^{2} \cdot 7
$$
Rō ràng bài toán yêu cầu đếm số thặng dư chính phương theo modulo 2016. Theo bổ dề 2 thì
$$
s(2016)=s\left(2^{5}\right) s\left(3^{2}\right) s(7)
$$
Theo bổ đề 1 thì
$$
s\left(2^{5}\right)=2^{2}=4, s\left(3^{2}\right)=\frac{3^{2}-3}{2}=3, s(7)=\frac{7-1}{2}=3
$$
Do đó, số các số $a$ cần tìm là $4 \cdot 3 \cdot 3=36$.

Bài 2.

1) Dự đoán dấu bằng xảy ra khi $a=1, b=2, c=3, d=4$ nên ta có các đánh giá sau
$$
\left\{\begin{array}{l}
a^{2}+1 \geq 2 a \\
b^{2}+4 \geq 4 b \\
c^{2}+9 \geq 6 c \\
d^{2}+16 \geq 8 d
\end{array}\right.
$$
Do đó, ta có
$$
\begin{aligned}
&24(a+b+c+d) \leq 3\left(d^{2}+16\right)+4\left(c^{2}+9\right)+6\left(b^{2}+4\right)+12\left(a^{2}+1\right) \\
&=3 d^{2}+4 c^{2}+6 b^{2}+12 a^{2}+120 \\
&=3\left(a^{2}+b^{2}+c^{2}+d^{2}\right)+\left(a^{2}+b^{2}+c^{2}\right)+2\left(a^{2}+b^{2}\right)+6 a^{2}+120 \\
&\leq 3 \cdot 30+14+2 \cdot 5+6 \cdot 1+120=240
\end{aligned}
$$
Suy ra $a+b+c+d \leq 10$.
2) Ta có:
$$
16 a^{2}+d^{2} \geq 8 a d \text { và } 9 b^{2}+4 c^{2} \geq 12 b c
$$
Từ đó suy ra
$$
\begin{aligned}
&24(a d+b c) \leq 3\left(16 a^{2}+d^{2}\right)+2\left(9 b^{2}+4 c^{2}\right) \\
&=3\left(a^{2}+b^{2}+c^{2}+d^{2}\right)+5\left(a^{2}+b^{2}+c^{2}\right)+10\left(a^{2}+b^{2}\right)+30 a^{2} \\
&\leq 3 \cdot 30+5 \cdot 14+10 \cdot 5+30 \cdot 1=240
\end{aligned}
$$
Suy ra $a d+b c \leq 10$.

Bài 3.

Goi $(*)$ là điều kiện đề bài cho. Trong $(*)$, thay $x=y=0$, ta có
$$
f(-2 f(0))=3 f(0)
$$
Đặt $f(0)=a$ thì $f(-2 a)=3 a$. Trong $(*)$, thay $x=0$ và $y=-2 a$, ta có
$$
f(-2 f(-2 a))=5 a-2 f(-2 a) \Leftrightarrow f(-6 a)=-a
$$

Trong $(*)$, thay $x=-2 a, y=-6 a$, ta có
$$
\begin{aligned}
&f(-2 a-2 f(-6 a))=5 f(-2 a)-4 x-2 f(-6 a) \\
&\Leftrightarrow f(0)=15 a+8 a+2 a \\
&\Leftrightarrow a=25 a \\
&\Leftrightarrow a=0
\end{aligned}
$$
Do đó $f(0)=0$.
Trong $(*)$, thay $y=0$, ta có
$$
f(x)=5 f(x)-4 x \Leftrightarrow f(x)=x
$$
Thử lại ta thấy thỏa.
Vậy hàm số cần tìm chính là
$$
f(x)=x, \forall x \in \mathbb{R}
$$

Bài 4.

1) Gọi $O$ là tâm của đường tròn $k$. Không mât tính tống quát, giả sử tia $\Lambda D$ nằm giữa hai tia $A O, A B$, các trường hợp còn lại tương tự.
Ta có:
$$
\angle I D B=\angle A B C, \angle I D C=\angle A C B
$$
nên
$$
\angle B A C+\angle B D C=\angle B A C+\angle A B C+\angle A C B=180^{\circ}
$$

Do đó, tứ giác $A B D C$ nội tiếp hay $D \in(O)$. Ta thấy
$$
\begin{aligned}
&\angle D A O+\angle O I D \\
&=\angle B A C-(\angle D A B+\angle O A C)+360^{\circ}-\left(90^{\circ}+\angle D I C\right) \\
&=\angle B A C-\left(\angle I C D+90^{\circ}-\angle A B C\right)+270^{\circ}-\angle D I C \\
&=\angle B A C+\angle A B C-(\angle I C D+\angle D I C)+180^{\circ} \\
&=\left(180^{\circ}-\angle A C B\right)-\left(180^{\circ}-\angle I D C\right)+180^{\circ} \\
&=\angle I D C-\angle A C B+180^{\circ}=180^{\circ}
\end{aligned}
$$

Do đó, AOID nội tiếp hay đường tròn $(A I D)$ di qua $O$ cố định.
2) Ta có:
$$
\angle E A C=90^{\circ}-\angle B A C, \angle F A B=90^{\circ}-\angle B A C
$$
nên
$$
\angle E A F=180^{\circ}-2 \angle B A C+\angle B A C=180^{\circ}-\angle B A C
$$
Do đó, góc $\angle E A F$ có số đo không đổi.

Bài 5.

Trước hết, ta chứng minh bổ đề sau:
Giá trị của biểu thức
$$
\frac{1}{1}+\frac{1}{2}+\frac{1}{3}+\cdots+\frac{1}{n}
$$
tiến tới vô cực khi $n \rightarrow+\infty$. Thật vậy,
Xét hàm số $f(x)=\ln (1+x)-x$ với $x>0$. Ta có
$$
f^{\prime}(x)=\frac{1}{1+x}-1<0
$$
nên đây là hàm nghịch biến, suy ra $f(x)<f(0)=0$ hay $\ln (1+x)<$ $x, \forall x>0$. Thay $x$ bởi $\frac{1}{n}$, ta được
$$
\ln \left(1+\frac{1}{n}\right)<\frac{1}{n} \Leftrightarrow \frac{1}{n}>\ln (1+n)-\ln n
$$
Do đó,
$$
\frac{1}{1}+\frac{1}{2}+\frac{1}{3}++\frac{1}{n}>\ln 2-\ln 1+\ln 3-\ln 2+\cdots+\ln (n+1)-\ln n=\ln (n+1)
$$
Vì $\ln (n+1) \rightarrow+\infty$ khi $n \rightarrow+\infty$ nên
$$
\frac{1}{1}+\frac{1}{2}+\frac{1}{3}+\cdots+\frac{1}{n} \rightarrow+\infty
$$
Trở lại bài toán, đặt
$$
y_{n}=\frac{x_{1}+x_{3}+x_{5}+\cdots+x_{2 n-1}}{x_{2}+x_{4}+x_{6}+\cdots+x_{2 n}}
$$
với $n \geq 1$. Ta thấy vì $\frac{1}{n} \in\left(0 ; \frac{\pi}{2}\right)$ nên $\cos \frac{1}{n}>0$, suy ra
$$
x_{n}=\frac{1}{n \cos \frac{1}{n}}>0, n \geq 1
$$

Xét hàm số $f(t)=\frac{t}{\cos t}$ với $t \in\left(0 ; \frac{\pi}{2}\right)$ thì $f^{\prime}(t)=\frac{\cos t+t \sin t}{\cos ^{2} t}>0$ nên đây là hàm đồng biến. Chú ý rằng $x_{n}=f^{2}\left(\frac{1}{n}\right)$, mà $\frac{1}{n}$ là dãy giảm nên $x_{n}$ cũng là dãy giảm.
Suy ra $x_{1}>x_{2}, x_{3}>x_{4}, \ldots, x_{2 n-1}>x_{2 n}$ nên $y_{n}>1$
Ngoài ra, ta cũng có $x_{3}<x_{2}, x_{5}<x_{4}, \ldots, x_{2 n-1}<x_{2 n-2}$ nên
$$
\begin{aligned}
y_{n}<& \frac{x_{1}+\left(x_{2}+x_{4}+\cdots+x_{2 n-2}\right)}{x_{2}+x_{4}+\cdots+x_{2 n}}=\\
& 1-\frac{x_{1}-x_{2 n}}{x_{2}+x_{4}+\cdots+x_{2 n}}<1-\frac{x_{1}}{x_{2}+x_{4}+\cdots+x_{2 n}}
\end{aligned}
$$
Dễ thấy rằng
$$
x_{2}+x_{4}+\cdots+x_{2 n}=\sum_{i=1}^{n} \frac{1}{2 i \cos \frac{1}{2 i}} \geq \sum_{i=1}^{n} \frac{1}{2 i}=\frac{1}{2} \sum_{i=1}^{n} \frac{1}{i}
$$
Theo bổ đề trên thì $\sum_{i=1}^{n} \frac{1}{i}$ tiến tới vô cực nên
$$
\lim \left(x_{2}+x_{4}+\cdots+x_{2 n}\right)=+\infty
$$
Do dó
$$
\lim \left(1-\frac{x_{1}}{x_{2}+x_{4}+\cdots+x_{2 n}}\right)=1-0=1
$$
Theo nguyên lý kẹp, ta có $\lim x_{n}=1$.

Bài 6.

Đặt $X=x-1, Y=y+1$, thay vào, ta có
$$
\begin{aligned}
&\left\{\begin{array}{l}
X^{2}+Y^{2}=b \\
Y-1=(X+1)^{2}+(2 a+1)(X+1)+a^{2}
\end{array}\right. \\
&\Leftrightarrow\left\{\begin{array}{l}
X^{2}+Y^{2}=b \\
Y=X^{2}+(2 a+3) X+a^{2}+2 a+3
\end{array}\right.
\end{aligned}
$$
Ta đưa về tìm điều kiện của $b$ để tồn tại $a$ mà hệ trên có nghiệm $(X, Y)$. Do
$$
Y-(X+2)=X^{2}+2(a+1) X+(a+1)^{2}=(X+a+1)^{2} \geq 0
$$

nên $Y \geq X+2$. Suy ra $Y-X \geq 2>0$, tức là $(X-Y)^{2} \geq 4$. Ta có
$$
b=X^{2}+Y^{2}=\frac{(X-Y)^{2}+(X+Y)^{2}}{2} \geq \frac{(Y-X)^{2}}{2} \geq 2
$$
Mặt khác, với $b \geq 2$, nếu chọn $X=-(a+1)$ thì có $Y=X+2=1-a$. Khi đó, ta có
$$
X^{2}+Y^{2}=(a+1)^{2}+(a-1)^{2}=2\left(a^{2}+1\right)=b
$$
Như thế, với $a$ thỏa mãn $2\left(a^{2}+1\right)=b$ thì hệ có nghiệm là
$$
(X, Y)=(-a-1,1-a)
$$
Dễ dàng thấy rằng do $b \geq 2$ nên luôn tồn tại $a$ như thế.
Vậy các giá trị cần tìm của $b$ là $b \geq 2$.

Bài 7.

1) Giả sử ngược lại, tồn tại 2 cặp $\left(A_{i}, B_{i}\right)$ và $\left(A_{j}, B_{j}\right)$ thỏa mãn điểu kiện đề bài đã cho.
Vì $i \neq j$ nên theo giả thiết,
$$
\left|A_{i} \cap B_{j}\right| \geq 1,\left|A_{j} \cap B_{i}\right| \geq 1
$$
Đặt $x_{r} \in A_{i} \cap B_{j}, x_{s} \in A_{j} \cap B_{i}$ với $1 \leq r, s \leq n$ thì:
– Do $x_{r} \in B_{j}$ nên với mọi $x_{k} \in A_{j}$, ta đều có $k<r$.
– Do $x_{r} \in A_{i}$ nên với mọi $x_{k} \in B_{i}$, ta đều có $k>r$.

Từ đây suy ra
$$
A_{j} \subset\left\{x_{1}, x_{2}, \ldots, x_{r-1}\right\}, B_{i} \subset\left\{x_{r+1}, x_{r+2}, \ldots, x_{n}\right\}
$$
Điều này cho thấy $A_{j} \cap B_{i}=\varnothing$, mâu thuẫn với giả thiết.
Vậy tồn tại không quá 1 cặp $\left(A_{i}, B_{i}\right)$ thỏa mãn điều kiện đã cho.
2) Gọi $T$ là tập hợp các cách chọn hai dãy
$$
A_{1}, A_{2}, \ldots, A_{m} \text { và } B_{1}, B_{2}, \ldots, B_{m}
$$
thỏa mãn điều kiện là: với mỗi $i, j \in\{1,2,3, \ldots, n\}, A_{i} \cap B_{j}=\varnothing$ nếu và chỉ nếu $i=j$.
Gọi $T_{i} \subset T$ là các cách chọn sao cho sao cho cặp $\left(A_{i}, B_{i}\right)$ thỏa mãn điều kiện là: cặp $\left(A_{i}, B_{i}\right)$ với $i=1,2,3, \ldots, n$ sao cho nếu $x_{k} \in A_{i}$ và $x_{l} \in B_{i}$ thì $x_{k}<x_{l}$ (ở đây ta xét thứ tự ban đầu của các phần tử của $X$ ). (*)
Theo câu 1) thì $T_{i} \cap T_{j}=\varnothing$ với $i \neq j$ nên ta có
$$
\left|T_{1}\right|+\left|T_{2}\right|+\cdots+\left|T_{m}\right|=\left|T_{1} \cup T_{2} \cup \ldots \cup T_{m}\right| \leq T
$$
Tiếp theo, với $1 \leq i \leq m$, xét một tập hợp $S \subset X$ và $|S|=a_{i}+b_{i}$. Khi đó, tương ứng với $S$, có đúng 1 cách chọn $\left(A_{i}, B_{i}\right)$ thỏa mãn tính chất $(*)$ – tức là $A_{i}$ sẽ nhận $a_{i}$ số nhỏ nhất trong tập $S, B_{i}$ là lấy phần còn lại.
Trong khi đó, nếu không có điều kiện $(*)$, ta có thể chọn tùy ý $C_{a_{i}+b_{i}}^{a_{i}}$ phần tử trong $S$ và $A$ và số còn lại cho $B$.
Do đó, ta có
$$
\left|T_{i}\right|=\frac{|T|}{C_{a_{i}}^{a_{i}}+b_{i}}
$$
với $i=1,2, \ldots, m$. Từ đây suy ra
$$
\sum_{i=1}^{m} \frac{|T|}{C_{a_{i}+b_{i}}^{a_{i}}} \leq|T| \Leftrightarrow \sum_{i=1}^{m} \frac{1}{C_{a_{i}+b_{i}}^{a_{i}}} \leq 1
$$
Ta có đpcm.

Bài 8.

1) Giả sử $E F$ cắt $B C$ ở $L$ và $(T),(O)$ cắt nhau tại $J$ khác $A$. Suy ra $A J$ chính là trục đẳng phương của $(T),(O)$. Do đó $O T \perp A J$.
Khi đó,
$$
L B \cdot L C=L E \cdot L F
$$
nên $L$ thuộc trục đẳng phương của $(T),(O)$. Suy ra $A, J, L$ thẳng hàng. Theo định lý Brocard cho tứ giác $B E F C$ nội tiếp trong đường tròn $(I)$ thì $I$ chính là trực tâm của tam giác $A D L$.
Vì thế nên ID $\perp A L$, mà $O T \perp A J$ nên $I D \| O T$.

2) Dễ dàng thấy rằng $D$ là trực tâm của tam giác $A G H$ nên $A D \perp G H$. Ta sẽ chứng minh rằng $A, D, K$ thẳng hàng.

Ta có $D B \cdot D F=D E \cdot D C$ nên $D$ có cùng phương tích tới 2 đường tròn $(A B F),(A E C)$. Suy ra $A D$ chính là trục đẳng phương của 2 đường tròn này.

Bằng biến đổi các góc nội tiếp, ta thấy rằng
$$
\angle M P Q=\angle M B F=\angle C E F=\angle C N Q
$$
Suy ra $M N P Q$ nội tiếp, dẫn đến $K M \cdot K P=K N \cdot K Q$, tức là $K$ cũng có cùng phương tích tới 2 đường tròn $(A B F),(A E C)$.
Từ đó suy ra $A, D, K$ thẳng hàng. Do đó, $D K$ vuông góc với $G H$.

Đề thi và đáp án tuyển sinh vào lớp 10 PTNK không chuyên 2015

Đề thi vào lớp 10 trường Phổ thông Năng khiếu năm 2015

Bài 1.

a) Giải phương trình $ \left( x^2 -9 \right) \sqrt{2-x}= x\left( x^2-9 \right) $

b) Giải hệ phương trình $\left\{ \begin{array}{l} \left( x^2 + 4y^2 \right) ^2 – 4\left( x^2 + 4y^2 \right) =5 \\ 3x^2 + 2y^2 =5 \end{array}\right. $

Giải

a) Điều kiện $2-x>0 \Leftrightarrow x \le 2$

$\left( x^2 -9\right) \sqrt{2-x}=x\left( x^2-9 \right)$

$\Leftrightarrow \left( x^2 -9 \right) \left( \sqrt{2-x}-x \right) =0$

$\Leftrightarrow \left[ \begin{array}{l} x=3 \ (l) \\ x=-3 \ (n) \\ \sqrt{2-x}=x \ (2)  \end{array}\right. $

Ta có $\sqrt{2-x}=x \Leftrightarrow \left\{ \begin{array}{l} x\ge 0 \\ 2-x=x^2 \end{array}\right. \Leftrightarrow x=1$

vậy $S=\left\{ -3;1 \right\} $

b) $\left\{ \begin{array}{l} \left( x^2 + 4y^2 \right) ^2 – 4\left( x^2 + 4y^2 \right) =5 \ (1)\\ 3x^2 + 2y^2 =5 \ (2) \end{array}\right. $

Đặt $t=x^2+4y^2$, $t\ge 0$ từ (1) ta có $t^2-4t-5=0 \Leftrightarrow \left[ \begin{array}{l} t=5 \ (n) \\ t=-1 \ (l) \end{array}\right. $

Ta có hệ $\left\{ \begin{array}{l} x^2+4y^2=5 \\ 3x^2+2y^2=5 \end{array}\right. \Leftrightarrow \left\{ \begin{array}{l} x^2=1 \\ y^2 =1 \end{array}\right. \Leftrightarrow \left\{ \begin{array}{l} x = \pm 1 \\ y= \pm 1 \end{array}\right. $

Vậy hệ phương trình có 4 nghiệm $(x;y)$ là $(1;1),(1;-1),(-1;1),(-1;-1)$

Bài 2. Cho phương trình $\dfrac{\left( x-2m \right) \left( x+m-3 \right) }{x-1}=0$ $(1)$

a) Tìm $m$ để phương trình $(1)$ có hai nghiệm phân biệt $x_1$,$x_2$.

b) Tìm $m$ để $x_1^2+x_2^2 -5x_1x_2= 14m^2 -30m +4$

Giải

a) $\dfrac{(x-2m)(x+m-3)}{x-1}=0$ $(1)$, điều kiện $x\ne 1$.

$(1) \Leftrightarrow \left[ \begin{array}{l} x=2m \\ x=3-m \end{array}\right. $

Phương trình $(1)$ có hai nghiệm phân biệt khi và chỉ khi

$\left\{ \begin{array}{l} 2m \ne 3-m \\ 2m \ne 1 \\ 3-m \ne 1 \end{array}\right. \Leftrightarrow \left\{ \begin{array}{l} m \ne 1 \\ m \ne \dfrac{1}{2} \\ m \ne 2 \end{array}\right. $

b) Theo câu a) thì điều kiện là $\left\{ \begin{array}{l} m \ne 1 \\ m \ne \dfrac{1}{2} \\ m \ne 2 \end{array}\right. $.

Giả sử $x_1 =2m, x_2=3-m$ ta có:

$x_1^2 + x_2^2 -5x_1x_2 = 14m^2 -30m +4 $

$\Leftrightarrow (2m)^2 + (3-m)^2 -5(2m)(3-m) = 14m^2 -30m +4 $

$\Leftrightarrow m^2 -6m +5 =0$ $\Leftrightarrow \left[ \begin{array}{l} m=1 \ (l) \\ m=5 \ (n) \end{array}\right. $

Bài 3.

a) Rút gọn biểu thức $Q= \left( \dfrac{3+\sqrt{x}}{3-\sqrt{x}}-\dfrac{3-\sqrt{x}}{3+\sqrt{x}}-\dfrac{36}{x-9} \right) : \dfrac{\sqrt{x}-5}{3\sqrt{x}-x}$ ($x>0, x\ne 9, x\neq 25$)

b) Tìm x để $Q<0$.

Giải

a) $Q= \left( \dfrac{3+\sqrt{x}}{3-\sqrt{x}}-\dfrac{3-\sqrt{x}}{3+\sqrt{x}}-\dfrac{36}{x-9} \right) : \dfrac{\sqrt{x}-5}{3\sqrt{x}-x} $

$= \left( \dfrac{\left( 3+\sqrt{x} \right) ^2-\left( 3-\sqrt{x} \right) ^2 +36}{\left( 3-\sqrt{x} \right) \left( 3+\sqrt{x} \right)} \right) \cdot \dfrac{\sqrt{x}\left( 3- \sqrt{x} \right) }{\sqrt{x}-5} $

$=\dfrac{12\sqrt{x}}{\sqrt{x}-5}$

b) Ta có $Q<0 \Leftrightarrow \dfrac{12\sqrt{x}}{\sqrt{x}-5}<0 \Leftrightarrow 0 < \sqrt{x} <5 \Leftrightarrow 0<x < 25 $.

So với điều kiện ta có: $\left\{ \begin{array}{l} 0<x<25 \\ x \ne 9 \end{array}\right. $

Bài 4.

a) Cho một tam giác vuông. Nếu ta tăng độ dài các cạnh góc vuông thêm $3$ cm thì diện tích tăng $33$ $cm^2$, nếu giảm độ dài một cạnh góc vuông $2$ cm và tăng độ dài cạnh góc vuông kia $1$ cm thì diện tích giảm $2$ $cm^2$. Hãy tính độ dài các cạnh của tam giác vuông.

b) Bạn An dự định trong khoảng thời gian từ ngày $1/3$ đến ngày $30/4$ mỗi ngày sẽ giải $3$ bài toán. Thực hiện đúng kế hoạch một thời gian, vào khoảng cuối tháng $3$ (tháng $3$ có $31$ ngày) thì An bị bệnh, phải nghỉ giải toán nhiều ngày liên tiếp. Khi hồi phục, trong tuần đầu An giải $16$ bài toán; sau đó, An cố gắng giải $4$ bài một ngày và đến $30/4$ thì An cũng hoàn thành kế hoạch đã định. Hỏi bạn An đã nghỉ giải toán ít nhất bao nhiêu ngày?

Giải

a) Gọi độ dài hai cạnh góc vuông là $x$, $y$ $(m)$. Theo đề bài ta có hệ phương trình:

$\left\{ \begin{array}{l} \dfrac{1}{2}(x+3(y+3) =\dfrac{1}{2}xy+33 \\ \dfrac{1}{2}(x-2)(y+1) =\dfrac{1}{2} xy-2 \end{array}\right. \Leftrightarrow \left\{ \begin{array}{l} x+y=19 \\ x-2y=-2 \end{array}\right. \Leftrightarrow \left\{ \begin{array}{l} y=7 \\ x=12 \end{array}\right. $

Độ dài cạnh huyền $z=\sqrt{7^2+12^2} =\sqrt{193}$

b) Số ngày dự định làm là $61$ ngày, số bài toán dự định làm là $3.61=183$

Gọi $x$ là số ngày làm theo dự định, y là số ngày nghỉ ta có $x \le 31 $.

Số ngày làm $4$ bài/ngày là $61 – x – y – 7 = 54 – x – y$ (ngày)

Theo đề bài ta có:

$$3.x+0.y+16+ 4(54-x-y) =183 \Leftrightarrow 4y+x=49$$

Mà $x \le 31 \Rightarrow 4y \ge 18 \Rightarrow y \ge \dfrac{18}{4}$, mà $y \in \mathbb{N}$ nên giá trị nhỏ nhất của $y$ là $5$.

Bài 5. Hình bình hành $ABCD$ có $ \angle ADC =60^\circ $ và tam giác $ACD$ nhọn. Đường tròn tâm $O$ ngoại tiếp tam giác $ACD$ cắt cạnh $AB$ tại $E$ ($E \ne A$), $AC$ cắt $DE$ tại $I$.

a) Chứng minh tam giác $BCE$ đều và $OI \bot CD$.

b) Gọi $K$ là trung điểm $BD$, $KO$ cắt $DC$ tại $M$. Chứng minh $A$, $D$, $M$, $I$ cùng thuộc một đường tròn.

c) Gọi $J$ là tâm đường tròn ngoại tiếp tam giác $ABC$. Tính $\dfrac{OJ}{DE}$.

Giải

a) Ta có $\angle BEC = \angle ADC =60^\circ $ ($ADCE$ nội tiếp) và $\angle ABC = \angle ADC = 60^\circ  $ ($ABCD$ là hình bình hành)

Tam giác $BCE$ có $\angle EBC =\angle BEC = 60^\circ  $ nên là tam giác đều.

Ta có $\angle DCE = 180^\circ  – \angle DAE =60^\circ $, suy ra $\angle DCE = \angle ADC$ nên hình thang $AECD$ là hình thang cân

Khi đó $\angle ACD = \angle EDC$, tam giác $ICD$ cân tại $I$.

Ta có $IC = ID$, $OC = OD$ nên OI là trung trực của $CD$. Do đó $OI \bot CD$

b) Ta có $K$ là trung điểm $BD$ nên $K$ cũng là trung điểm $AC$ do $ABCD$ là hình bình hành

Khi đó $OK \bot AC$ và OK là trung trực của AC. Suy ra$MA=MC$. Suy ra $\angle MAC = \angle ACM$.

Mà $\angle ACM = \angle IDM $

Từ đó $\angle IDM = \angle MAC$. Suy ra tứ giác $AIMD$ nội tiếp.

c) Ta có $JK \bot AC$. Suy ra $I$, $K$, $O$ thẳng hàng. Do tam giác $ABC$ và tam giác $ACD$ bằng nhau nên $JK = OK$.

Mặt khác $\angle KJC = \dfrac{1}{2} \angle AJC = \angle ABC =60^\circ  $

Khi đó $\dfrac{KJ}{CK}=\cot \angle KJC = \dfrac{1}{\sqrt{3}}$

Mà $OJ=2JK$, $DE=AC$ ($AECD$ là hình thang cân), $OK=\dfrac{1}{2}AC$.

Khi đó $\dfrac{OJ}{DE} = \dfrac{2JK}{AC}=\dfrac{2JK}{2CK}=\dfrac{KJ}{CK}=\dfrac{1}{\sqrt{3}}$

Vậy $\dfrac{OJ}{DE}=\dfrac{1}{\sqrt{3}}$

 

 

Đáp án đề thi học kì 1 môn toán lớp 10 trường Phổ thông Năng khiếu năm 2016

Bài 1. Tìm m để phương trình $\dfrac{(x-1)(x-3m)}{\sqrt{x-2}+1}=0$ vô nghiệm

Bài 2. Gọi $(P)$ là đồ thị của hàm số: $y= x^2 + bx + c \, \, (b,c \in \mathbb{R} )$. Biết các điểm $A(1;-4)$, $B(2;-3)$, thuộc $(P)$. \
Tìm tọa độ giao điểm của $(P)$ và $(P’)$, với $(P’)$ là đồ thị của hàm số $y= (2x-1)^2 -4$

Bài 3. Cho hệ phương trình: $\left\{ \begin{array}{l}
x+\dfrac{1}{m} \sqrt{y} =4 \\
\dfrac{1}{m} x + \sqrt{y} = \dfrac{2}{m} + 2
\end{array} \right.$, với m là tham số và $m \ne 0$.

Định m để hệ phương trình có nghiệm duy nhất.

Bài 4. Giải các phương trình sau:

a) $\sqrt{2x+1}+\sqrt{x-3}=4$
b) $x+ \dfrac{3x}{\sqrt{x^2-9}}=\dfrac{35}{4}$

Bài 5. Chứng minh đẳng thức: $\tan^2 a – \tan^2 b = \dfrac{\sin(a+b).\sin(a-b)}{\cos^2a.\cos^2b}$

Bài 6. Cho tam giác $ABC$ có các đỉnh $A(-1;3)$, $B(-3;-3)$, $C(2;2)$. Chứng minh tam giác $ABC$ là tam giác vuông và tìm trực tâm tam giác $ABC$.

Bài 7.  Cho hình bình hành $ABCD$ với $AB=6a$, $AD=3a$, $\angle ABC =60^0$. Gọi $M,N$ thỏa: $\overrightarrow{MA}+2 \overrightarrow{MB}=\overrightarrow{0}$, $3 \overrightarrow{ND}+2 \overrightarrow{NC}=\overrightarrow{0}$.
a) Tính $\overrightarrow{AM}. \overrightarrow{AD}$.
b) Tính độ dài cạnh $AN$ theo $a$.
c) Gọi $G$ là trọng tâm tam giác $AMN$. Tìm $x$ và $y$ thỏa: $\overrightarrow{BG}= x \overrightarrow{BA} + y \overrightarrow{BD}$.

Hết

Đáp án

[userview]

ptnk10hk12016

[/userview]

Đề thi và đáp án tuyển sinh vào lớp 10 TPHCM 2015

I. ĐỀ tuyển sinh vào lớp 10 TPHCM 2015

Bài 1. Giải các phương trình và hệ phương trình sau:
a)  $x^2 – 8x+15=0$.
b)  $2x^2 – \sqrt{2}x -2 =0$.
c)  $x^4 -5 x^2 -6=0$.
d) $2x+ 5y = -3$ và $3x-y =4$

Bài 2.

a) Vẽ đồ thị (P) của hàm số $y = x^2$ và đường thẳng (D): $y = x + 2$ trên cùng một hệ trục tọa độ.
b) Tìm tọa độ giao điểm của (P) và (D) ở câu trên bằng phép tính.

Bài 3. Thu gọn các biểu thức sau:
a) $A = \dfrac{{\sqrt x }}{{\sqrt x – 2}} + \dfrac{{\sqrt x – 1}}{{\sqrt x + 2}} + \dfrac{{\sqrt x – 10}}{{x – 4}}\left( {x \ge 0,x \ne 4} \right)$
b) $B = (13-4\sqrt{3})(7+4\sqrt{3})-8\sqrt{20+2\sqrt{43+24\sqrt{3}}}$.
Bài 4. Cho phương trình $x^2 – mx+m-2=0$ (1) ($x$ là ẩn số).

a) Chứng minh phương trình (1) luôn có 2 nghiệm phân biệt với mọi giá trị $m$.
b) Định $m$ để hai nghiệm $x_1, x_2$ của (1) thỏa $\dfrac{x_1^2-2}{x_1-1}.\dfrac{x_2^2-2}{x_2-1} = 4$.
Bài 5. Cho tam giác $ABC$ có $AB < AC$ có ba góc nhọn. Đường tròn tâm $O$ đường kính $BC$ cắt các cạnh $AC, AB$ lần lượt là tại $E, F$. Gọi $H$ là giao điểm của $BE$ và $CF$. D là giao điểm của $AH$ và $BC$.
a) Chứng minh $AD \bot BC$ và $AH.AD = AE.AC$.
b) Chứng minh $EFDO$ là tứ giác nội tiếp.
c) Trên tia đối của tia $DE$ lấy điểm $L$ sao cho $DL = DF$. Tính số đo góc $BLC$.
d) Gọi $R, S$ lần lượt là hình chiếu của $B, C$ lên $EF$. Chứng minh $DE + DF = RS$.

II. ĐÁP ÁN

Bài 1. Giải các phương trình và hệ phương trình sau:
a) $x^2 -8x +15 =0$
$\Delta ‘ =1 $
Hai nghiệm của phương trình là $x_1 = 3$; $x_2 =5$
b)  $2x^2 – \sqrt{2}x -2 =0$
$\Delta =18$
Hai nghiệm của phương trình là $x_1 = \sqrt{2}$; $x_2 = \dfrac{-\sqrt{2}}{2}$
c) $x^4 – 5x^2 -6 =0 $
Đặt $t= x^2 \ge 0$
Phương trình trở thành $t^2 -5t -6=0$
$\Delta = 49$
$t_1 = -1$ (loại) và $t_2 = 6$ (nhận)
Với $t=6 \Rightarrow x= \pm \sqrt{6}$
d) $2x+ 5y =-3 \;\; (1)$ và $3x-y =4 \;\; (2)$
$\Leftrightarrow  2x+5y = -3 \;\; (1) $ và $17x = 17 \;\; ((1) + 5\cdot (2))$
$ \Leftrightarrow  x=1 $ và $y= -1$.

Bài 2.

a) Lưu ý: $(P)$ đi qua $O(0;0)$, $( \pm 1 ;1)$, $\pm 2; 4 )$
$(D)$ đi qua $(1;3)$, $(0;2)$

Đồ thị:
b) Phương trình hoành độ giao điểm của $(P)$ và $(D)$ là:
$x^2 = x + 2 \Leftrightarrow x^2 -x-2=0 $

$\Leftrightarrow  x=-1$ và $x=2$
$y(-1) = 1$, $y(2)=4$
Vậy tọa độ giao điểm của $(P)$ và $(D)$ là $(2;4)$, $(-1;1)$.

Bài 3.
a) $A=\dfrac{\sqrt{x}}{\sqrt{x}-2}+ \dfrac{\sqrt{x}-1}{\sqrt{x}+2}+ \dfrac{\sqrt{x}-10}{x-4} $
$= \dfrac{\sqrt{x} \left( \sqrt{x}+2 \right) + \left( \sqrt{x} -1 \right) \left( \sqrt{x}-2 \right) + \sqrt{x}-10}{x-4} $
$= \dfrac{x+2\sqrt{x}+x – 2\sqrt{x}-\sqrt{x}+2+ \sqrt{x}-10}{x-4}
= \dfrac{2x-8}{x-4} =2$
b) $B=\left( 13- 4\sqrt{3} \right) \left( 7+ 4\sqrt{3} \right) – 8\sqrt{20 + 2\sqrt{43 + 24\sqrt{3}}} $
$= 43 + 24\sqrt{3} – 8 \sqrt{20 + 2\sqrt{\left( 13-4\sqrt{3} \right) \left( 7+4\sqrt{3} \right) }} $
$= 43 + 24\sqrt{3} – 8\sqrt{\left( \sqrt{13-4\sqrt{3}} + \sqrt{7+ 4\sqrt{3}} \right) ^2} $
$= 43 + 24\sqrt{3} -8 \left( \sqrt{\left( 2\sqrt{3}-1 \right) ^2} + \sqrt{\left( 2+ \sqrt{3} \right) ^2 } \right) $
$= 43 + 24\sqrt{3} – 8 \left( 3\sqrt{3}+1 \right) $
$=35$.
Bài 4.

a) $x^2 – mx +m-2 =0$ $(1)$
$\Delta = m^2 -4m +8 = (m-2)^2 + 4 >0, \; \forall m$
Do đó phương trình $(1)$ luôn có hai nghiệm phân biệt.
b) Theo Viet, ta có:

$S= x_1 + x_2 = m $ và $P = x_1 \cdot x_2 = m-2$
$\dfrac{x_1^2 -2}{x_1-1} \cdot \dfrac{x_2^2-2}{x_2-1} =4 $
$\Leftrightarrow x_1^2x_2^2 – 2\left( x_1^2 + x_2^2 \right) + 4 = 4x_1x_2 – 4 \left( x_1 + x_2 \right) +4 $
$\Leftrightarrow P^2 -2 \left( S^2 -2P \right) -4P + 4S =0 $
$\Leftrightarrow P^2 -2S^2 + 4S =0 $
$\Leftrightarrow (m-2)^2 -2m^2 + 4m =0 $
$\Leftrightarrow -m^2 +4 =0 $
$\Leftrightarrow m= \pm 2$
Cách khác:
$x_1$, $x_2$ là hai nghiệm của phương trình nên:
$x_1^2 -mx_1 +m -2=0 \Rightarrow m= \dfrac{x_1^2-2}{x_1-1}$
$x_2^2 -mx_2 +m-2 =0 \Rightarrow m= \dfrac{x_2^2-2}{x_2-1}$
$\dfrac{x_1^2-2}{x_1-1} \cdot \dfrac{x_2^2 -2 }{x_2-1} =4
\Leftrightarrow m^2 =4 \Leftrightarrow m= \pm 2$.
Bài 5.


a) $\angle BEC = \angle BFC =90 ^\circ $
$H$ là trực tâm của $\triangle ABC \Rightarrow$ $AD$ là đường cao của $\triangle ABC \Rightarrow AD \bot BC$.
$\triangle ADC \backsim \triangle AEH \Rightarrow AH \cdot AD = AE \cdot AC$.
b) $\angle EOC = 2\angle EFC $
Tứ giác $HFBD$ nội tiếp $\Rightarrow \angle CFD = \angle EBC$ mà $\angle EBC = \angle CFE$
$\Rightarrow \angle CFD = \angle CFE \Rightarrow \angle DFE = 2\angle CFE$
Suy ra: $\angle EOC = \angle DFE \Rightarrow$ tứ giác $EFDO$ nội tiếp.
c) $EFDO$ nội tiếp $\Rightarrow \angle EDF = \angle EOF = 2\angle FCE$ (1)
Tam giác $DFL$ cân tại $D$ $\Rightarrow \angle EDF = 2\angle FLE$ (2)
Từ (1) và (2) $\Rightarrow \angle FCE = \angle FLE$
$\Rightarrow$ $EFLC$ nội tiếp $\Rightarrow L \in (O) \Rightarrow \angle BLC =90^\circ $
d) $\angle BIC =90^\circ \Rightarrow $ $SRBI$ là hình chữ nhật $\Rightarrow RS= BI$ (3)
$DF = DL$ và $OF = OL \Rightarrow $ $OD$ là trung trực của $FL$
$\Rightarrow \angle BIL = \angle BEF$ (vì cung $BL$ và $BF$ bằng nhau)
Mà $\angle BEF = \angle EBI$ nên $\angle BIL = \angle EBI \Rightarrow BE // LI$
$\Rightarrow $ $BEIL$ là hình thang cân $\Rightarrow EL = BI$ (4)
Từ (3) và (4) $\Rightarrow EL = RS$ hay $DE + DF = RS$.

 

 

Đáp án toán PTNK 2015

Bài 1. (Toán chung) Hình bình hành $ABCD$ có $ \angle ADC =60^0$ và tam giác $ACD$ nhọn. Đường tròn tâm $O$ ngoại tiếp tam giác $ACD$ cắt cạnh $AB$ tại $E$ ($E \ne A$), $AC$ cắt $DE$ tại $I$.
a. Chứng minh tam giác $BCE$ đều và $OI \bot CD$.
b. Gọi $K$ là trung điểm $BD$, $KO$ cắt $DC$ tại $M$. Chứng minh $A$, $D$, $M$, $I$ cùng thuộc một đường tròn.
c. Gọi $J$ là tâm đường tròn ngoại tiếp tam giác $ABC$. Tính $\dfrac{OJ}{DE}$.

Gợi ý

Bài 2. (Toán chuyên) Cho tam giác $ABC (AB < AC)$ có các góc nhọn, nội tiếp trong đường tròn tâm $O$. Gọi $M$ là trung điểm của cạnh $BC$, $E$ là điểm chính giữa của cung nhỏ $BC$, $F$ là điểm đối xứng của $E$ qua $M$.
a. Chứng minh $EB^2 = EF.EO$.
b. Gọi $D$ là giao điểm của $AE$ và $BC$. Chứng minh các điểm $A, D, O, F$ cùng thuộc một đường tròn.
c. Gọi $I$ là tâm đường tròn nội tiếp tam giác $ABC$ và $P$ là điểm thay đổi trên đường tròn ngoại tiếp tam giác $IBC$ sao cho $P, O, F$ không thẳng hàng. Chứng minh rằng tiếp tuyến tại $P$ của đường tròn ngoại tiếp tam giác $POF$ đi qua một điểm cố định.

Gợi ý

a.

  • Ta có $E$ là điểm chính giữa cung BC, suy ra $EB = EC$ và $OE \bot BC$ nên $M, O, E$ thẳng hàng.
  • Vẽ đường kính $EK$. Ta có $EM.EK = EB^2$.
  • Mặt khác $EF = 2EM, EO = \dfrac{1}{2}EK$. Do đó $EF.EO = EM.EK = EB^2$. (1)

b.

  • Ta có $\angle EBC = \angle EAC = \angle EAB$. Suy ra $\Delta EAB \sim \Delta EBD$. Suy ra $EB^2 + ED.EA$ (2).
  • Từ (1) và (2) ta có: $EA.ED = EO.EFF$. Suy ra tứ giác $OFDA$ nội tiếp.

c.

  • Ta có $\angle EIB = \angle EAB + \angle ABI = \dfrac{1}{2}(\angle A + \angle B) = \angle EBC + \angle CBI = \angle EBI$, suy ra $EB = EI = EC$. Vậy $E$ là tâm đường tròn ngoại tiếp tam giác $BIC$. Do đó $EP = EB$. Ta có $EP^2 = EB^2 = EO.EF$.
  • Suy ra $\Delta EPF \sim \angle EOP$. Suy ra $\angle EPF = \angle FOP$.
  • Hơn nữa, do $O,F$ cùng phía đối với $E$ nên $PO, PF$ cùng phía đối với $PE$.
  • Vẽ tia tiếp tuyến $Px$($PF, PO$ cùng phía đối với $Px$)của đường tròn ngoại tiếp tam giác $POF$. Khi đó $\angle xPF = \angle FOP = \angle EPx$. Suy ra $Px$ và $PE$ trùng nhau. Vậy $Px$ luôn qua điểm $E$ cố định.